Этого треда уже нет.
Это копия, сохраненная 18 ноября 2019 года.

Скачать тред: только с превью, с превью и прикрепленными файлами.
Второй вариант может долго скачиваться. Файлы будут только в живых или недавно утонувших тредах. Подробнее

Если вам полезен архив М.Двача, пожертвуйте на оплату сервера.
image.png157 Кб, 350x272
Тред тупых ответов и умных вопросов. 481090 В конец треда | Веб
ТРЕД ТУПЫХ ВОПРОСОВ И УМНЫХ ОТВЕТОВ ЗДЕСЬ

Прошлый:>>476364 (OP)
https://2ch.hk/sci/res/476364.html (М)
2 481094
У меня умных вопросов нет, да и тупых тоже. Просто бампаю тред.
3 481095
>>479403
Надо сначала понять, бывают ли частоты вне нотного спектра. Мне кажется такого нет, потому что когда ноты заканчиваются, они начинаются снова, на новой высоте. И так до бесконечности, хз.
4 481101
Раз уж тут дают умные ответы, может вы разгадаете эту советскую загадку, не могу уже час понять, что там внутри "
Летит самолет, в нем ежик с ним коробка, самолет упал и разбился. Ежик умер. Что было в коробке"
5 481118
Есть материал который не гнется.
Ну вот чтобы взять палку длиной 100 метров, держать её вертикально а они что б сука не прогнулась ни на мм.
Ощущение что даже лист стекла 1 метр если держать за край на весу горзонтально дугой становится. какого хуя, геде пропрыв в материалах обезьяны бля, космос вам, хуй вам а не косомос материловедение курите для начала.
6 481119
>>1118

>держать её вертикально


Горизонтально, сори.
7 481120
>>1118
А нафига ощущения, вот тебе деформация 100 метровой балки.
29 метров вертикально вниз.
Причём выбрана самая мощная на рынке в каталоге балка высотой 1метр (высота сечения)
glas.jpg184 Кб, 1783x699
8 481122
>>1118
Вот для примера стекло (высота сечения листа 1 м, ширина 30 см)

У него E modul ещё меньше, так что оно прогнётся на 42 метра.
9 481130
>>1118
Даже идеальные материалы погнутся должны. Мир очень рыхлый на больших масштабах, надежда только на какую-нибудь экзотическую материю из недр нейтронных звезд, но это невозможно воссоздать в лабораторных условиях никак и никогда.
10 481131
>>1130
Такая материя схлопнется в чёрную дыру под своим весом, если будет в сильном гравитационном поле хотя бы даже той же земли.

Те расчёты выше это манямир, там нагрузка такая на балку, что она просто не выдержит, но если бы выдержала, то погнулась бы как в этих расчётах.

С балкой из нейтронной звезды не лучше, она весит так много, что под своим весом в месте наивысшего момента на изгиб в поле гравитации прогнулась бы так, что образовалась бы чёрная дыра и опять бы ничего не вышло.

Только если материал не имеет массу и прочный, но это невозможно вообще никак, даже теоретически.
11 481136
>>1131

>Такая материя схлопнется в чёрную дыру под своим весом, если будет в сильном гравитационном поле хотя бы даже той же земли.


Очевидно, что нет. Недостаточно массы.

>С балкой из нейтронной звезды не лучше, она весит так много, что под своим весом в месте наивысшего момента на изгиб в поле гравитации прогнулась бы так, что образовалась бы чёрная дыра и опять бы ничего не вышло.


А если балка будет с сечением в ангстрем?
12 481142
>>1136
Не знаю, посчитай сам.
13 481163
ЧТо значит 671?
Озон 671
Битард 671
Кто-то ещё 671
14 481165
Математики, а лучше математики-программисты, поясните, пожалуйста:
дискретное фурье преобразование и дискретное косинусное преобразование нуждаются в нормализации или нет?
15 481177
Что такое пространство?
16 481179
>>1177
Сериал, на 7/10 где-то
17 481194
>>1118
Потому что всё держится не жёстко, а на электромагнитном взаимодействии.

Всё гуляет.
А дохера жёсткое и не нужно, закон квадрата-куба всё равно не даст огромных йоб построить, хоть усрись. Это тебе не мультики. Всё будет таких размеров, каких позволяют фундаментальные законы этой вселенной.
18 481196
>>1177
Область, заполненная полями, благодаря чему в ней может что-то происходить.
19 481197
Что важно для нагрева предмета(посуды, куска материала) на индукционной плите? Или вернее что важно для хорошей работы связки плита-предмет/посуда.
Удельное сопротивление материала, или сопротивление сечения?

В интернете везде по этому поводу какой-то бред про магнитится-не магнитится, но мне кажется что это даунские бредни маркетологов и попытка по-простому дать юзерам возможность отличить некоторые материалы у которых большое сопротивление, от некоторых тех у чего маленькое.
20 481204
>>1177
Совокупность точек, между которыми можно задать соотношения расстояния.
21 481207
>>1197
Какой же ты тупой, пиздец.
Суть индукционной плиты в том, что оно создает локальные сильные вихревые токи, которые постоянно перемагничивают магнитные материалы(ферро- и ферри- магнетики, т.е то что хорошо магнитится - почти любая сталь), которые уже перевод энергию в тепло. Желательно чтоб материал вообще ток не проводил, потому что скин-эффект будет вытеснять ток на поверхность, сильно снижая эффективность. Поэтому алюминиевую или медную посуды индукционная печь будет греть тонкий внешний слой (а если ты совсем уебан может появится дуга), в случае немагнитных материалов с плохой проводимостью, эффективность будет еще ниже.
22 481208
>>1197
Для того, чтобы эффективно тратить энергию вихревых токов дно посуды должно быть толстым (чтобы равномерно греть) и иметь большое сопротивление (чтобы быстро энергию на тепло переводить, этим же максимальная мощность посуды ограничена, чем меньше она тем дольше будет греть один и тот же обьем). Стальной блин неплохо подходит. Технически индукционка может (и будет) греть любой проводник, даже простой карандаш.
Насчет магнита ты прав - это простой способ для людей определить, норм посуда или нет, без вкапывания в дебри физики. Важно и сечение и удельное сопротивление, вместе.
23 481209
>>1208
а хуйню пизданул в общем
24 481238
>>1177
Множество с дополнительными структурами. Векторное пространство, топологическое, метрическое и т.д.
25 481241
>>1101
Сначала подумал, что пассажирский лайнер летел и в грузе был ёжик с коробкой, но ежей вроде никогда нельзя было так транспортировать, а если всё-таки можно, тогда в коробке был ёж, потому что коробка была с ежом.

А если самолёт был мелкий, из этой коробки, то в коробке схема сборки и запчасти от этого самолёта.

Скорее всего, в любом случае, в коробке ёжик, потому что она с ним. Но это не точно.
SetZh.jpg36 Кб, 400x290
26 481243
Есть ли какая то взаимосвязь с относительностью времени в ОТО, принципом гейзенберга в квантовой механике и идеей о n-мерности времени?

Почему ОТО постулирует, что время должно течь иначе у разных наблюдателей, когда можно так же постулировать, что каждый наблюдатель живёт в своём временном измерении?

Почему Принцип Гейзенберга говорит, что мы не можем узнать сразу две характеристики, если другая характеристика просто живёт в другом времени, а до измерения обе характеристики эволюцинируют в двух временах (после измерения одна характеристика навсегда уходит в другой время), то есть волновая природа, это волны вызванные от взаимодействия ОДНОГО электрона с самим собой, но в двухмерном времени.

Почему нет годной теории на этот счёт ещё, это ведь я додумался даже: анон на мамкиной шее начитавшийся научпопа.
27 481246
>>1243
Нету никакой взаимосвязи, эти два принципа абсолютно о разном говорят.
Ты вообще неправильно понимаешь идею о многомерном времени. Если у тебя знания на уровне "научпопа начитался", то я думаю тебе вообще не следует влезать в эту гипотезу. Потому что многомерное время это просто научные фантазии ученых, разминка для ума, жонглирование формулами. На сегодняшний день, нету никаких сомнений, что временная размерность наблюдаемой вселенной равна 1. Вся ОТО построена на математике псевдориманова многообразия с одним отрицательным собственным значением метрического тензора, которое соответствует временноподобному направлению. Разумеется можно эту метрику дополнить еще несколькими независимыми отрицательными тензорами (в таком случае и получается многомерное время). Но в таком случае все уравнения ОТО принимают другой вид, и соответственно будут давать другие решения, которые не согласуются с наблюдаемой реальностью.

>Почему нет годной теории на этот счёт ещё, это ведь я додумался даже


Под галюциногенами я и не до такого додумывался. Вот только в науке все эти "додумки" имеют мало ценности. Выкладывай давай строгое математическое описание своих додумок, с пояснениями и авторитетными рецензиями. Только тогда твои додумки будут чего то стоить.
Spacetimedimensionality.svg.png103 Кб, 1024x1024
28 481247
29 481249
>>1246
А в чём разница между моим пониманием и правильным пониманием?

Не надо в ОТО добавлять просто другое измерение конечно, это не будет работать. ОТО же это не фундмантельная теория, а просто некая маняфантазия, которая случайно описывает какие то узкие явления.
Надо создавать новую теорию, на базе предложенного мной выше аргумента, но так как в математике я только синус умею на кулькуляторе считать, то давай ты это сделаешь.
30 481250
>>1246

>1. Вся ОТО построена на математике псевдориманова многообразия с одним отрицательным собственным значением метрического тензора, которое соответствует временноподобному направлению.


1. ОТО невременизависима, той в ней время просто какой то фон, как было в классике и квантовой.
2. Время не вытекает из фундмантельных оснований, не получается из более простых принципов, а вводится как фон искусственно руками. В итоге природа времени непонятна. Какие то коэффициенты, хуй короче а не теория.
3. Постулируется относительность времени, но не объясняется фундаментально каким образом у каждого наблюдателя свою время. Просто как отче наш.
31 481254
Правда ли существуют устройства, позволяющие управлять компьютером с помощью мыслей?
32 481255
>>1177
Набор элементарных мест под материю, связанных комбинаторными правилами перемещения и взаимодействия материи.
33 481256
>>1254
Полноценно - нет
Вроде максимум, до куда дошли - теребить указатель
34 481257
>>1243
Нет, совсем нет.
Время для наблюдателяСобственное время в ОТО это длина некоторой кривой в некотором пространстве. Вообще время в классических теориях поля это расстояние между двумя точками в пространстве событий. Движение тут понимается некоторый 1-мерный объект, составляющийся из композиций(взаимных нагромождений) различных разрешенных(в рамках теории) преобразований. В добавок движение наделено дифференциальной структурой - или гладкостью, это означает, что на достаточно малом масштабе преобразования линейны. Эти бесконечно малые преобразования позволяют выделить на фоне первичного пространства создать вторичное пространство, подобное первому. Объединим первичное и вторичное пространство получим фазовое пространство, которое содержит особую математическую структуру. Эта структура подобная проводнику, которая проводит нас из начальной точки в другую ЕДИНСТВЕННЫМ путем - наше искомое движение или 1- мерный объект. И длина этого пути(куска движения) и есть собственное время.
В квантовой теории у нас есть две полевые системы, у них разная мера ("объем" их фазового пространства). Как бы не хотели, но прямо состыковать или смешать две системы нельзя, но можно их проецировать на друг друга. Принцип Гейзенберга это размер "окна" для взаимного проекции двух систем. И время тут опять длина между двумя связанными точками. Т.е. Для твоего сраного НАБЛЮДАТЕЛЯ это по прежнему расстояние между двумя событиями.
В обоих теориях время это длина или расстояние между двумя залупами. Если ввести комплекснозначное(двумерное как бы, но за то как бы еще "число") собственное время это полностью разрушить теорию(точнее делает бессмысленным структуру в фазовой пространстве, все дело в том, что перепрыгнуть в мнимую часть и наоборот гладким движением нельзя), породив нежизненноспобного монстра. Впрочем время как пространственно-временную координату можно задать комплексным числом, для этого надо построить 8 мерное пространство с ебанутыми преобразования, которые излишни, но расстояние все равно должны быть выражены действительным числом(или чисто мнимым, это как считать).
Ах, да. Есть топологические ограничения на рабочие структуры с движением.
34 481257
>>1243
Нет, совсем нет.
Время для наблюдателяСобственное время в ОТО это длина некоторой кривой в некотором пространстве. Вообще время в классических теориях поля это расстояние между двумя точками в пространстве событий. Движение тут понимается некоторый 1-мерный объект, составляющийся из композиций(взаимных нагромождений) различных разрешенных(в рамках теории) преобразований. В добавок движение наделено дифференциальной структурой - или гладкостью, это означает, что на достаточно малом масштабе преобразования линейны. Эти бесконечно малые преобразования позволяют выделить на фоне первичного пространства создать вторичное пространство, подобное первому. Объединим первичное и вторичное пространство получим фазовое пространство, которое содержит особую математическую структуру. Эта структура подобная проводнику, которая проводит нас из начальной точки в другую ЕДИНСТВЕННЫМ путем - наше искомое движение или 1- мерный объект. И длина этого пути(куска движения) и есть собственное время.
В квантовой теории у нас есть две полевые системы, у них разная мера ("объем" их фазового пространства). Как бы не хотели, но прямо состыковать или смешать две системы нельзя, но можно их проецировать на друг друга. Принцип Гейзенберга это размер "окна" для взаимного проекции двух систем. И время тут опять длина между двумя связанными точками. Т.е. Для твоего сраного НАБЛЮДАТЕЛЯ это по прежнему расстояние между двумя событиями.
В обоих теориях время это длина или расстояние между двумя залупами. Если ввести комплекснозначное(двумерное как бы, но за то как бы еще "число") собственное время это полностью разрушить теорию(точнее делает бессмысленным структуру в фазовой пространстве, все дело в том, что перепрыгнуть в мнимую часть и наоборот гладким движением нельзя), породив нежизненноспобного монстра. Впрочем время как пространственно-временную координату можно задать комплексным числом, для этого надо построить 8 мерное пространство с ебанутыми преобразования, которые излишни, но расстояние все равно должны быть выражены действительным числом(или чисто мнимым, это как считать).
Ах, да. Есть топологические ограничения на рабочие структуры с движением.
35 481258
>>1257
Анонче, ты разбираешься к квантах?
Не знаешь, как можно описать функцию плотности вероятности наблюдения для частицы, сидящей в яме, которую смещают на dx?
36 481259
>>1258
Не совсем тебя понял, ты хочешь сдвинуть всю систему, только яму, или посмотреть на симметрию?
37 481260
>>1259
Я хочу сдвинуть именно яму вдоль одной оси.
38 481261
>>1260
*За очень короткий промежуток времени.
39 481262
>>1261
Тогда у тебя просто сама плотность съедет, в чем проблема, че тебе конкретно нужно?
40 481263
>>1257
Забудь сейчас про постановку в ото или квантмех, очевидно это уже пробовали попробуй описать все с нуля. Видно что ты шаришь, у тебя получиться в отличии от меня

Смотри я буду забрасывать а ты записывай математически.
3 местное пространство и n мерное время (искривление пространства времени забудь).
У каждой системы свое время зависящее от силы гравитации и скорости.
Гравитация это лишь проявление некой силы между разными временами (искажение зависит от разности времени, больше масса, разное время, то есть время зависит от массы). Ускоренное движение это путешествие между n изменениями времен изменениями времени.

Если мы всю ОТО запишем не как кривизну пространства времени а как плоское эвклидовое 3 мерное пространство + n - пространственное время - то святая математика которая конечно всегда должна быть права (может математика врёт?) то что будет?
Чиста попробовать
Я так не умею так что давай ты
41 481265
>>1263

>то есть время зависит от массы


Не от массы, в уравнении ОТО массы нет.
42 481267
>>1254
С дивана в калавдути играть не выйдет. Сейчас все на уровне таскания курсора (стрелочками) и нажатия нескольких функциональных кнопок. Все это медленно, неточно.
43 481270
>>1265
Забудь ото
44 481271
>>1263

>n мерное время


Ты не понимаешь что такое время и что такое размерность.
Впрочем, что с дебила и шизика взять.
45 481277
>>1271
Не в рамках ОТО
Блядь, ты слышишь меня?
46 481278
Слушай, что ты не понимаешь в n мерном времени? Тут же понимать блядь нечего.

Система 1: позиция: xyz, скорост s1, масса m1, время t1, ускорение a1
Система 2: позиция: xyz, скорост s2, масса m2, время t2, ускорение a2

Итого пространство 3 мерное
скорость многомерная
масса - скаляр, фактор
время многомерное

что сложного, бля?
47 481279
>>1278
А философском варианте:
каждая система живёт в своём времени
ты в своём, я в своём
мы эволюцинируем по разному, потому что у нас разная скорость и разная масса (живём в разных системах). У нас у каждого своё время. У тебя одно, у меня другое.
Спутник на орбите ещё другое время.

И не подставляй в ОТО, сука, не трогай, ОТО. Оставь, не трогай, мамка наругает.
48 481281
В какой лаборатории в ДС можно проверить вещество (не запрещенное, не лекарственное) нормально недорого и без заебов.
Если не знаете лабораторию, то скажите где можно спросить?
49 481284
>>1279
>>1278
Вот поэтому философов выгнули из академической науки, лол.

С таким походом к описаю мира, система будет не рабочая, совсем нахуй. Как ты будешь стыковать манямирки систем? Как со своим особенным временем в полученном пространстве совершать преобразования?
50 481287
>>1284
Это следующий вопрос.
Ты ведь даже не пробывал стыковывать, а уже "нахуй".
Чё ты так.

Я думаю хуйню предложил, но разве сотни других теорий, типо струнной лучше?
51 481293
>>1287
Всякие физические теории не на голом месте строятся, есть некоторые фундаментальные обобщения, вокруг которых и пляшут. Теории строятся не по принципу, а давай от балды добавим хрень и посмотрим что выйдет. Теории используют простые обобщения и обобщения из них строят теорию.
Лагранжева механика механика родилась из ньютоновской при анализе диффур. В ней переформируется движение как некий объект в некотором пространстве. Законы поведения объекта уже задаются через экстремальные принципы (принцип наименьшего действия). Все последующие теории(от максвелловской электродинамики до теории струн) в той или иной мере используют такой подход.
И вот приходишь ты и говоришь свои маняфантазии, даже без должной формулировки. Ты даже толком не разобрался с концептом времени как характеристики движения и требуешь вещи, которые нельзя вывести из базовых обобщений.
52 481302
>>1293
Но сегодня это не так, как раз придумывают от балды всякие суперсимметрии, струны ебать, этож смешно.
53 481303
>>1293
И основная проблема ещё, что теоретики ищует красивую математику, а не реальную физику.

А мы знаем, что неизведанное недоступно интуитивному опыту и не обязано никак состыковываться с нашими прошлыми моделями мира, оно может быть вообще любым, вплоть до того что мелкие рептилиоиды микромира сидят и клешнями двигают электроны играя в дурака, а когда это не так, то рептилоид ПРОСТО ПОШУТИЛ
54 481312
>>1302
Суперсимметрия это первых шаг к попытки обобщению всех полей. Довольно последовательных шаг.
Струны внезапно родились из рабочих теории сильных взаимодействий. Позже выяснилось, что можно через них скрещивать другие поля с другом другом.
Ничего от балды не придумывали.
55 481313
>>1312
Да, только для всего этого нужно было выдумать несуществующие частицы, и толи 10 толи 11 измерений, и проблема выбора ландшафта, то есть это по сути теория, когда х любое число. Абсолютно ничего на дающая теория. Эпици

Теория растянутых резинок это. Неужели ты думаешь мир так работает?
56 481314
>>1313
Эпициклы какие то, а не теория.

И только глаза струнных горят

МАТЕМАТИКА ТАКАЯ КРАСИВАЯ МАТЕМАТИКА СМОТРИ СХОДИТСЯ ТАХИОНЫ СОКРАЩАЮТСЯ СМОТРИ СМОТРИ

Ну пиздец, с чего они взяли что это должно быть КРАСИВО?

В итоге 40 лет - нихуя
Коллайдер построили - нихуя.

Не ну чё, всё как бы верно делаете.
57 481346
1.Если клетки могут лишь в ограниченное число делений, то часто травмируемый человек быстрее состарится?
2.100ваттная неубиваемая лампочка, разогретая до температуры солнечного ядра, будет светить на 10^16Вт?
58 481348
59 481349
>>1314

>Ну пиздец, с чего они взяли что это должно быть КРАСИВО?


Потому что математика это так себе замаскированный мистицизм, и когда звёзды сходятся, это очень тешит.
60 481351
>>1346

>2.100ваттная неубиваемая лампочка, разогретая до температуры солнечного ядра, будет светить на 10^16Вт?


Или она в таком случае не стоватная, раз смогла разогреться до стольки, имея размеры обычной стоватной лампы, или, если таки стоватная, то область, разогретая до температуры солнечного ядра в ней будет такая маленькая, что она будет светить на 100 ватт. Интенсивно, но с оооочень маленькой площади/поверхности.
Или, её что-то разогрело до такой температуры, или она сама, но она была укрыта чем-то что возврощало обратно ей всё её излучение и она ничего не излучала во вне, в результате чего разогрелась, долго работая не светя. Она будет доли секунды светить намного больше 100 ватт, но будет быстро остывать по убывающей экспоненте, пока не станет светить на свои 100 ватт. Если будет включена.
61 481387
Когда уже замутят нанороботов, которые изнутри фиксят человека, что мешает их замутить?
Когда создадут там бананы, чтоб их можно было в сибири выращивать?
62 481388
>>1387
И что мешает замутить такие бананы/любые тропические фрукты
63 481389
>>1388
Строй теплицу, чтобы морозы и короткий солнечный день не мешал, да и выращивай. Еще при царе это делали.
64 481392
Сап, сайэнс. Подскажи научных трудов, талмудов, исследований БЕЗ политического/феменистического/анти-феменистического подтекста на тему исследования разницы между мужчинами и женщинами? С точки зрения функциональных различий, а не "мужчины в среднем выше женщин и поэтому лучше".

Хочу максимально непредвзято на это взглянуть и для докладов полезно. Спасибо!
65 481394
>>1392
Я тоже интересовался этой проблемой.
Просто смотри научные исследования через ссылки на вики и гугли.

Я пришел к выводу что разницы в интеллекте нет, маленькие локальные флуктуации нивелируются индивидуальностью каждого человека. По крайней мере отчетливого консенсуса по поводу различий нет, разные работы иногда показывают отличия в интеллекте, но все это не подтверждается или опровергает я другими работами. Почему то в области 3 медного вращения объектов есть небольшая корреляция, что тянки с этим справляются хуже. И вообще задачи на навигацию позиционирование.

Но что есть и чем реально обусловлена разница - в культуре и воспитании. Например, мой личный опыт уверен что мужчины хуже в эмоциональном интеллекте и социальном взаимодействии, хуже справляются с чередой множественных задач, зато лучше могут вникнуть в какую то одну область и достигнуть в ней вершины.
Но это все западное, зависит как ты будешь воспитывать и какое влияние будет оказывать общество.
Например, я люблю детей и семью и не против свадьбы и всей этой девичьей романтики а моя тянка больше мечтает о карьере и жизненных достижениях. Просто на меня огромное влияние оказала в детстве мама, а на неё дедушка.
Так что разница между тян кун - это больше про общество, которое из нас делает тян и кунов, а не про физиологию. Разные гормональные баланса и тайминги да, но это не влияет так как культура и воспитание.
66 481401
>>1389
Хочу чтобы мутанты на морозе росли
67 481409
>>1394

>эмоциональном интеллекте


Почему не стоит тебя после такого окрестить долбоёбом и не попросить проследовать нахуй?
68 481412
>>1401
Чекай скорость химических реакций в зависимости от температуры.
69 481416
>>1394
Спасибо за размышления! Если вдруг есть какая-то конкретная статья, которая тебя особенно заинтересовала — дай знать, пожалуйста!

Оригинальный задавальщик вопроса
70 481427
>>1409
Потому что хуй занят, на нём вертятся все учоные, которые когда-либо пытались сформулировать определение интеллекта.
71 481489
>>1409
Эмоциональный интеллект на работу на должности руководителя проектов спрашивают на уровне тех. компетентности. Это очень важный элемент, влияющий на продуктивность человека.
Так как мы социальные существа и прогресса добились лишь благодаря командной работе, умение понимать эмоции других и адекватно на них реагировать - неотъемлимая часть. Речь даже не в том, как ты сумеешь договориться с людьми, а в том, как ты сможешь договориться с собой. Например, гениальный в математике человек, но неумелый в эмоциональном интеллекте, загорится какой-то идеей, не совладает с внутренними импульсами и потратит всю жизнь на "теорию поля", умерев в забвении, а другой соберёт команду и создаст "стандартную модель". Технические знания не помогут тебе, если ты не умеешь управлять своимы абмициями, эго, понимать других, и самое главное уметь делать себя счастливым, чтобы продуктивно работать.
72 481583
А что, не получится сделать дирижопль, чтоб оболочка была сплошь покрыта фотоэлементами и питала двигатель?
73 481585
>>1583
Нах надо+уже есть.
image.png462 Кб, 728x503
74 481599
>>1583
Ты не поверишь, он уже есть и даже не особо дорогой.
75 481604
Я как-то где-то видел краем глаза на википедии что бытие существует 9 миллионов раз (была точная цифра) в секунду. Это называлось толи моментом/мгновением/излучением. Или это была просто какая-то максимально минимальная единица измерения времени, напомните плиз, не могу нагуглить.
76 481618
>>1604
Максимальная малая единица времени, имеющая значение - планковское время. Меньше уже совсем другие истории и само понятие времени теряет смысл.
77 481619
>>1618
>>1604
sci который мы заслужили
78 481626
>>1618
Нет.
79 481630
>>1619
>>1626
Лан, я обосрался. Бахните свое объяснение.
80 481631
Верно ли, что нефти на планете ещё овермегадохуя, в т.ч. в нашей бензоколонке, в т.ч. более-менее легкодобываемой?
81 481632
>>1631
Да. Нефть имеет абиогенное происхождение и ее очень дохуя в недрах и она постоянно нарождается снова, возгоняясь из мантии.
83 481647
>>1631
Смотря что называть нефтью.
Самая легкодобываемая и самая "мейнстримная" это законсервированные трупики наших предков.
Впрочем в мантии оказалась дохуя водорода и углерода, а еще кремний и алюминий хорошо на себя кислород принимают у водорода. При определенных условиях в мантии образуется область низкой плотности, которая быстро заполняется углеродом и водородом. Получается этакий "пузырек", который сплывает к коре, в определенный момент "пузырек" может лопнуть внутрь (из-за движение плит) и "залиться" в промежуточный слой коры, если к нему есть доступ. Остывая там происходят процессы синтеза углеводородов. Хоть процесс медленный, но мантия срет пузыриками миллиарды лет и вполне уже есть слои, содержащую абиогенную нефть, на допустимой на нас глубине. Только такая нефть более рассеянная и глубокая, и методом "ткнуть палку в землю и сама польется "не достанешь.
84 481657
А воспоминания раннего детства (до 3 лет) всё - физически, напрочь стёрты? А насчёт 4+ лет?
85 481670
Поясните пжл для начинающего, но пока несведущего анона - в чем собственно заключается противоречие между ОТО и квантовой механикой?
86 481693
>>1657

>А воспоминания раннего детства (до 3 лет) всё - физически, напрочь стёрты


Никак не могут быть, они основа твоего подсознательного. Другое дело, что "ты" никакого доступа к ним не имеешь. Как, скажем, процессор пекарни не имеет прямого доступа к линии 220 вольт, хотя ею питается.
87 481720
>>1670
Противоречий там нет.
Дело в том, что ОТОшная гравитация как поле не квантуется. В смысле его можно взять и попробывать, но выходить хуита на первых этапах.
Почему так выходить? Это очень долгий разговор.
Впрочем ОТО все же научись квантовать различными способами, вот только какой способ правильный рассудить эксперимент, который мы не можем провести.
88 481722
Сап двач. Вопрос таков: если я встану на горизонт событий черной дыры, увижу ли я свою спину впереди?А если учитывать, что чд - шар, увижу ли я сам себя в каждой точке пространства вокруг себя на расстоянии окружности черной дыры? Будет ли для меня горизонт событий бесконечной плоскостью?
89 481724
>>1722
Все зависит от приливных сил и вращается ли ЧД, но в принципе да, только вот ты будешь в фотонной сфере, и единственное что ты увидишь это ярко голубую полосу, на этом фоне спину хуй рассмотришь.
90 481725
>>1720
а я читал, что согласно ОТО, Вселенная в момент Большого взрыва должна была быть точкой нулевого радиуса. А это запрещено принципом неопределенности. Так схуя ли радиус должен быть нулевым? Почему нельзя просто в качестве начального условия указать конечный радиус, чтобы всем стало хорошо?
91 481727
>>1725
Так так и делают уже
92 481728
>>1090 (OP)
доброе утро треду! вопрос такой: хочу снова вкатиться в математику (как хобби). в универе была на 1 и 2 курсе, но я уже ничего не помню. с чего посоветуете начать?
93 481733
>>1725
Слишком много писать. Проблема космологической сингулярности гораздо в высших ипостаси находиться нежели ОТО и квантовая теория. Другими словами обе теории не применимы к таким вещам.

>Так схуя ли радиус должен быть нулевым?


Интерполяция. ОТО геометрическая теория непрерывного говна в первую очень и должна выполнять геометрические правила или сохранять топологию. Если вселенная расширилась и есть начальный момент, то по топологическим правилам образуется разрыв или сингулярность. Чтоб закрыть эту срань, устремляют все к нулю.

>А это запрещено принципом неопределенности.


Принципом неопределенности этого не запрещает, он всего лишь устанавливает границы отображения двух систем, причем на произвольной фоновой геометрии, это кстати одна из причин невозможности классического квантования гравитации как поля.

>Почему нельзя просто в качестве начального условия указать конечный радиус, чтобы всем стало хорошо?


А как выбрать этот радиус? Нельзя просто так все подогнать, иначе пиздец. Впрочем многие этим пользуются с закономерным результатом.

Ну еще майндфак напоследок, в момент Большего взрыва Вселенная была как одна неразделимая физическая система, тогда как коллапс ее волновой функции вообще мог произойти?
94 481734
>>1728
Определись с направлением, а вообще пиздуй в /math, там целый тематический тред для таких дебилов.
95 481736
>>1734
понял, принял
96 481739
>>1733

>Если вселенная расширилась и есть начальный момент, то по топологическим правилам образуется разрыв или сингулярность


Поправь меня если я ошибаюсь, но начальный момент - он для нас начальный, т.к. мы не можем ничего сказать про состояние вселенной до большого взрыва. Но это не значит что до него - ничего не было. Просто никаких свидетельств о том состоянии не осталось, и на нынешнее состояние Вселенной то состояние не влияет, поэтому мы ничего об этом не можем о нем сказать.
97 481744
>>1739
Хуй знает как донести мысль.
Короче геометрия пространства-времени непрерывна, причем строгая, у этого пространства есть топология, которая требует сохранения определенных величин. С нашим пространством времени и наблюдаемой топологий может быть только два момента, либо вселенная вечно существует(а то хуита, но другим причинам), либо вселенная началась с 0-ой точки. А чтоб топологию не нарушать, надо все в 0 уравнять. События до этого момента являются уже пустым множеством, если будет продолжать координату времени назад, то упрешься в ноль.
98 481751
Ребята, но времени же вообще не существует.
99 481757
>>1751
Тебя тоже не существует, и чего?
100 481761
>>1757
Ну и пошел нахуй тогда
101 481765
>>1761
Нет ты.
15677159516780.png362 Кб, 700x700
102 481774
Поясните, почему так бывает. Большинство порно видосов вообще не возбуждает, а иногда даже не в порно, а просто в клипе случайно сосок промелькнет, и шишка уже в небеса, стены ебать хочется. Как это работает?
image.png65 Кб, 658x662
103 481791
>>1090 (OP)
Как найти нули периодической функции?
sage 104 481797
>>1791
А как найти страну на карте?
105 481810
Если при приближении к скорости света у тела увеличивается масса - значит, и притягивать оно начинает всё сильнее?
106 481812
>>1797
тя ебёт???
107 481815
>>1797
Ладно, поправляюсь, как АНАЛИТИЧЕСКИ найти нули периодической функции?
108 481817
>>1815
Молодец.
Решить уравнение f(x) =0
image.png865 Кб, 1310x1054
109 481823
>>1817
Но ведь когда функция периодическая там же вроде всё по другому решается ибо нулей у неё сколько кайф может быть
110 481824
>>1810
Это такой троллинг?
Нет, масса это скаляр, инвариант, она не зависит от скорости и систем отсчета. В ОТО масса это вообще интеграл по объему, и если у нас пространство односвязное (никаких тебе кротовых нор и варпа, пидор), то она еще не зависит как мы считаем этот интеграл.
111 481830
>>1823
Зависит от функции, обычно задания сделаны таким образом, что можно установить периодичность через которую нули повторяются.
image.png167 Кб, 303x418
112 481840
>>1830
Понял-принял, всех благ
113 481846
По поводу неизменности скорости света. Правильно я понимаю, что если поставить фонарик на тележку и пустить ее ехать по направлению к детектору, то свет достигнет детектора именно со скоростью света, то есть скорость тележки не будет с ней суммироваться? А если тележка едет в обратном направлении с произвольной скоростью, будет ли скорость тележки вычитаться из скорости света?

А если допустим, два фотона в момент времени t=0 начали удаляться друг от друга в противоположном направлении - значит ли это что в системе отсчета на одном фотоне, второй будет удаляться от него со скоростью 2с?
114 481849
Чем сильнее напряжение тем сильнее магнитное поле?
Электроны заставляет двигаться магнитное поле?
115 481860
>>1849
Нет.
Нет.

В какой-нибудь ебучей система отсчета ответ на твой вопрос.
Да
Да
116 481861
>>1824
А что тогда увеличивается у движущегося с v->c тела в системе отсчёта наблюдателя? Ведь у тел с лоренц-факторами 10 и 1000 скорости почти не отличаются, а энергии и импульсы - пиздецки как. Или я сто и ото смешиваю?
117 481862
>>1846
Не, там релятивистская механика начинается, не все так просто. Вкратце - не, скорость света так не наебать.
118 481864
>>1861
Увеличивается энергия. Это раньше говорили мол "масса увеличивается", но это просто тупо НЕ У ДОБ НО так считать, так как тело в движении имеет разную инерцию по движению и под углом 90 градусов к нему.
119 481872
>>1861
Чтоб понять, что там только компоненты относительно друг друга увеличиваются надо переходить 4-вектора.
В физике движение это более абстрактная структура, энергия и импульс зависят от свойств пространства в которых определенно движение, а не тупо умножаем массу на скорость.
В пространстве-времени все тела движутся со скоростью света.
120 481873
>>1846
Где вы только учите СТО?

>в системе отсчета на одном фотоне


В этой системе отсчета расстояния вырождаются(изотропный вектор же) и все будет движется со скоростью света, никакого суммирования нахуй.
121 481874
Чем по своей природе может являться телекинез у представителя биологической формы жизни? Ну, вот мозг может испускать какие-то материальные силовые поля, да? Это ведь дохуя энергии нужно. Чем могут быть такие поля?
122 481875
>>1751
Так и есть. Есть только материя, чувак, который собрал часы, и стадо философов которое верит во время. Вера во время по сути та же вера, что и в религии. Нигде даже и намека нет на такую систему как "время". Есть материя, которую можно замедлять гравитационным и термическим воздействием, а также ускорять (расплавить стеклянную бутылку, ну чем не перемещение бутылки в будущее? Бутылка бы и так стекла и распласталась по полу при комнатной температуре за несколько тысяч лет).
123 481876
>>1874
Только если на пердачной тяге.
Мозг может испускать только тепловое излучение и шум ебаный, который слабее наводки от розетки питания.
Так что единственное что остается, что тело может производить достаточную пердачную тягу для фиксации объекта. Шутки шутками, но некоторые рыбы могут особыми движениями плавников создавать турбулентные течения, блокирующие маневры своих жертв.
124 481877
>>1876
Ну офигенно блин. То есть я даже пофантазировать не могу на тему телекинетиков. Разве что на тему андроида, в которого встроены гравитационные излучатели им регулируемые, ну или энергия ци еще.
125 481878
>>1877
Не слушай того долбаеба. Дрочи кванты и будет тебе счастье.
ййй.png18 Кб, 1244x462
Анон, я правильно понимаю? 126 481882
Анон, я правильно понимаю корпускулярно-волновой дуализм? В фотонах с течением времени происходит нечто имеющее периодичность. И если мы регистрируем в одной точке фотоны, то в этом потоке будет волновое изменение этой периодичности. Таким образом поток частиц при замере показывает волну?
ййй2.png26 Кб, 1244x462
127 481883
вот доделал пикчу
128 481884
Тогда становится логичным то что поток частиц ведет себя как волна. Это ведь абстрактная волна. Не думаю что свет в самом деле как вода на море из таких вот волн состоит.
129 481886
Получается электромагнитная волна это продольная, а не поперечная волна
все тот же шизик
130 481889
>>1884

> свет в самом деле как вода на море из таких вот волн состоит


именно что из таких же - это доказывается эффектами интерференции, дифракции и наличием дифракционного предела в микроскопии - из за того, что свет - это волна, не получается сфокусировать пучок света точнее, чем амплитуда его колебания
131 481890
>>1889

>интерференции


Ну это не противоречит моей картине. Да в "широком" потоке фотонов может быть поперечная волна между частицами. Я же имею ввиду гипотетический поток фотонов толщиной в один фотон, в котором не может возникнуть поперечная волна, но нечто похожее на продольную волну вполне может быть. Тоесть в потоке фотонов происходит нечто что со стороны может описываться и как поперечная волна и как что то похожее на продольную волну в потоке фотонов толщиной в один фотон.
132 481891
Интересно было бы провести опыт когда щель сужается и посмотреть как поведет себя спектр вследствие этого.
133 481892
>>1882

> в одной точке


если бы мы могли обойти принцип неопределенности Гейзенберга и могли изучать объекты, размером с фотон, то да,

> поток частиц при замере показывает волну



Корпускулярно волновой дуализм проще рассматривать на электронах - у них есть масса.

С одной стороны - электрон - это частица - она может лететь и пробивать своей кинетической энергией, скажем, фольгу если бы он был волной, он бы прошёл сквозь материал

С другой стороны - электрон - это волна - и он проявляет дифракцию если бы он был частицей, то дифракционной картины не наблюдалось
ййй2.png16 Кб, 777x292
134 481893
>>1889
вот я немного переделал свою картинку и здесь есть как поперечные так и продольные абстрактные "волны"
135 481894
>>1774
не твой - вот и бесишься
серьёзно, когда ты не видишь всей картины, ты достраиваешь в воображении образ, который тебе нравится. Поэтому тебе и юбки нравятся больше, чем порево
image.png259 Кб, 800x600
136 481895
>>1891
Увидишь дифракцию
137 481896
>>1892

>Корпускулярно волновой дуализм проще рассматривать на электронах


Электроны отрицательно заряжены, а значит должны отталкиваться друг от друга, фотоны заряда не имеют.
maxresdefault.jpg203 Кб, 1280x720
138 481897
>>1893
Хотя полностью чтобы показать мою картинку лучше подходит вот эта спиральная модель, в моей картинке не хватает четвертого потока, который должен быть за вторым.
image.png32 Кб, 300x197
139 481898
>>1890

> поток фотонов толщиной в один фотон, в котором не может возникнуть поперечная волна


в таких условиях фотон не сможет быть поперечной волной. Как только он попадёт в такую трубку - он поглотится. Кстати, такой эксперимент уже проведён - вся материя состоит из молекул, между атомами которой есть расстояние. Но фотон не может пройти между атомами - потому что он жирный и поперечная волна не пролазит - вот и поглощается
бтв, у фотона нет толщины
140 481899
>>1896
От этого электроны не теряют своих дуалистических свойств
141 481900
>>1898
тоесть если сделать дырку через которую пролезет только гамма-луч и пустить на дырку весь спектр - пройдет только гамма-луч, а все остальное отрежется? тогда получается что свет это как облако такое лол, которое на может существовать без малейшего кусочка себя.
122112.png21 Кб, 1255x495
142 481901
>>1898
может ли в таком случае существовать отдельное "облако" из фотонов, или фотоны могут существовать только в потоке себе подобных и если исчезнет источик - исчезнут уже вылетевшие из источника фотоны?
143 481902
>>1900

> тоесть если сделать дырку через которую пролезет только гамма-луч и пустить на дырку весь спектр - Пройдет только гамма-луч, а все остальное отрежется


Да, именно так и работает рентгеновский аппарат. Через тело проходят только маленькие рентгеновские лучи, тогда как большие лучи из видимого спектра поглощаются

> тогда получается что свет это как облако такое


Да, белый свет - это облако из фотонов с различной энергией и частотой, и их разнообразие такое же, сколько чисел умещается между 0 и 1.
144 481903
>>1882
>>1883
Нет, не правильно.
Короче. Фотон это набор возбуждения обычно привычного нам ЭМ поля, каждый элемент который дает в сумме некоторое дискретное знание, которое привязано к квантуемой величине. Такую хуиту называют волновым пакетом или "порцией" поля. Когда детектор/наблюдатель/твоя жирная мамаша-шлюха/тепловая система наблюдает фотон, рандомно берется из набора один элемент, который выглядит для наблюдателя как классическая ЭМ волна, да еще с нужной порцией энергии. Собственно этот элемент и есть результаты измерения.
Колебания и волны это все еще классическое теория. Из-за математической магии чтобы самосогласованно и красиво упаковать весь возможный набор волн, нужно ввести особую функцию, ее и называют волновой функцией. В некотором роде волновая функция наследует свойства своего аргумента. Чтобы это увидеть надо выходить в пространство состояний, вводить операторы для это пространстве, считать интегралы. На одном рисунке не покажет так. Но волновая функция не описывает некоторую волну в классическом смысле.
>>1886
С хуя? ЭМ волны продольны только в среде.
145 481904
>>1903
неплохое пояснение спасибо
146 481905
>>1901
Есть эксперименты, где и отдельные фотоны регистрировали. Получается, фотоны самодостаточны.
На твоей картинке, ты немного не так понимаешь волну. Мы можем сказать, какая длина волны - в твоём случае она в пару метров, но мы не можем определить амплитуду (высоту). Можно лишь сказать, что она достаточно невелика, чтобы корпускула света летела прямо. И если перерисовать твою картинку, то длина волны будет 2 метра, а высота в долю атома.

Посмотри статью, тебе будет интересно https://habr.com/ru/post/158161/

> исчезнут уже вылетевшие из источника фотоны


Нет. На их создание уже была потрачена энергия. Если бы они исчезли - нарушился бы принцип сохранения энергии. Вот и получается, что чтобы исчезнуть, им нужно передать свою энергию какой-то материи.
147 481906

>в треде глупых вопросов запретили вопросы о корпускулярно волновом дуализме и ОТО


>тред умер нахуй


добавьте ответы в шапку и разгрузите тредю
148 481907
>>1906
Пора уж ликбез для даунов пилить.
149 481908
>>1905

>а высота в долю атома.


А как еще называется высота эм-волны? Гуглил и везде только длину предлагают
150 481909
>>1908
>>1905

> мы не можем определить амплитуду (высоту)


Твой вопрос вызывает вопросы
151 481912
>>1909
Ну а почему бы не попытаться её определить?
152 481913
Что сейчас на фронте облачных технологий нового? Слышал, что через годик гугл выкатит какую-то йобу, заменяющую видяху по подписке. Правда чи не?
153 481917
>>1882

>я правильно понимаю корпускулярно-волновой дуализм?


хуле его понимать, в зависимости от того, какую задачу ты рассматриваешь, тебе удобнее ПРЕДСТАВЛЯТЬ себе либо частицу, либо волну. Рассматриваешь дифракцию - для тебя электрон это волна, рассматриваешь атом водорода - для тебя он частица, которая находится в потенциале другой частицы, почти неподвижной по сравнению с электроном. А в действительности это ни то ни другое, это нечто, что представить гораздо труднее, и поэтому прибегают к таким мысленным костылям, как частица и волна.

Нужно помнить, что понятия "частица" и "волна" придуманы людьми, чтобы худо-бедно описать явления, гораздо сложнее чем эти мысленные образы. В силу того что человек - обезьяна, только недавно оторвавшаяся от банана, наш мозг не приспособлен к восприятию каких-то вещей, которые лежат далеко за рамками нашего бытового восприятия. Попробуй, например, представить себе число миллион. Не "миллион чего-то", а просто Миллион. Хотя ты и миллион "чего-то" представить не сможешь. А представь себе число 10^24? Нихуя не получится. С электроном та же тема, это объект, образ которого плохо поддается нашему восприятию. Поэтому и придумали костыли.
154 481921
>>1913
Стадия чтоль? Ну, такие проекту уже лет десять пилят, но они подыхают один за другим, хз получится ли у гугела
155 481922
>>1917
это в шапку при перекате
ЯСНО?!
156 481929
>>1882
Фотоны как частицы лучше вообще не трогай. Мозги поломаешь, они ебучие, релятивисткие хуйни, которые даже положения не имеют и онли направление и поляризацию.
157 481931
>>1929
А по мне фотоны довольно простые объекты с минимум параметров. А спектральное разложение вполне интуитивно. Это тебе не электрон с ебучим спином.
sage 158 481934
>>1931
У фотона та же хуйня, только в виде поляризации.
SpinOne-Half(Slow).gif860 Кб, 250x250
159 481935
>>1931

>ебучим спином


Недавно на википедии выложили весьма адекватное пояснение спина.
160 481936
>>1934
Лол, поляризация же тривиальная хрень даже в классическом поле. Тем более задается унитарной группой.
sage 161 481937
>>1936
В чём тогда твоя проблема со спином?
162 481938
>>1937
Короче спин как таковой алгебраическая хуита по сути без явного геометрического представления. Долго въезжать. Для этого надо лесть в топологию.
163 481941
>>1938

>Для этого надо лесть в топологию.


Чего, блять? Шкил, ты б хоть постеснялся.
164 481942
>>1935
Всмысле недавно? Я несколько лет назад смотрел, вроде нихуя не изменилось.
165 481943
>>1877
Можешь ещё чё-то на тему тёмной энергии пофантазировать, эффект схожий, а засечь мочёные её не могут, кроме как по фактическим действиям от неё на тела.
166 481945
>>1860
Что тогда заставляет двигаться электроны и почему они движутся тем "сильнее" чем выше напряжение?

Понятно что напряжение заставляет, но это просто пустое слово, абстракция, что это такое?
167 481946
>>1945
Тебе учебник по электродинамике пересказать, или ты напишешь, что это тоже "ыыы абстракция"?
168 481947
>>1946
Если набирать не надо, а просто скопировать-да, скопируй момент в котором это поясняется.
Или если там слишком то своими словами коротко, если можно.
169 481948
>>1945
Сила лоренца.
170 481949
>>1947
В классике электрон движется в равновесии силы Лоренца и аналога силы лобового сопротивления, которая линейна по скорости электрона. Из условия равновесия этих сил получается закон Ома. В квантах похожие выводы следуют из физкинетики или гринистики, а для свободных электронов во внешних постоянных однородных полях есть известные решения.
171 481953
>>1949
>>1948

>Сила Лоренца – сила, действующая на точечную заряженную частицу, движущуюся в магнитном поле.


Т.е. увеличить напряжение это значит увеличить силу лоренца?
Так всё-таки магнитное поле значит увеличить, потому что на другие её составляющие не повлиять.

Кстати не понял почему среди них скорость движения частицы, ведь она, скорость движения, получается от силы лоренца, а не сила лоренца от движения частицы.
Хотя 1)это одно и то же, 2)двигаться может не сама частица, а источник поля или проводник в котором эта частица.

Но я-то спрашиваю не про такой случай, а просто про лежащий провод на концы которого подано напряжение.
Безымянный.jpg29 Кб, 850x231
172 481954
Из чего следует уравнение Шрёдингера? Везде, куда ни глянь, оно просто постулируется как некий фундаментальный закон, но если так - каким массивом экспериментов оно подтверждается? Как к нему пришли?
173 481958
>>1953

>Т.е. увеличить напряжение это значит увеличить силу лоренца?


да
А дальше ты что-то бессвязное написал.
174 481961
>>1958
Всё там норм.

Сила лоренца от силы магнитного поля, значит увеличить напряжение-увеличить силу магнитного поля?
175 481962
>>1954
Постулируется не уравнение, а форма гамильтониана. Само уравнение выражает общий вид динамической группы квантовых систем, а выбор гамильтониана подтверждается всеми экспериментальными следствиями квантмеха.
176 481963
>>1961
Магнитное поле тут причем? Тебе эффект Холла нужен?
177 481965
>>1963

>Она равна произведению заряда, модуля скорости частицы, модуля вектора индукции магнитного поля и синуса угла между вектором магнитного поля и скоростью движения частицы.


А нет, нет там в составляющих силы магнитного поля.

Мне нужно почему электроны двигаются при наличие напряжения, и почему они двигаются сильнее при увеличении напряжения.
Т.е. что такое напряжение.
178 481969
>>1965
Напряжение - разность потенциалов электростатического поля. Для однородного постоянного электрического поля она линейна по напряженности.
1501494596430.png5 Кб, 694x495
179 481983
>>1090 (OP)
Физики в треде есть? Поясните почему "гантель" пикрил не поедет через всю вселенную.

Гантель состоит из двух частей, одна — шар с плотностью меньше чем плотность среды, другая — шар с плотностью большей чем у среды. На тяжелый шар действует гравитация со стороны среды и легкого шара, но со стороны легкого шара она меньше. Поэтому он начинает убегать от легкого шара, но одновременно тянет его с собой. Получается движение без реактивных двигателей?

inb4: вселенная заполнена вакуумом, легкий шар не из чего сделать.
Но в вакууме есть звезды и планеты, так что если сделать шары астрономических размеров и вынуть из легкого шара все звезды и планеты, его плотность будет меньше средней плотности вещества во вселенной.
180 481984
>>1945
Классика.
Заряженные частицы "генерирует" поток заряда, который растекается по всему пространству-времени. Растекание потока неравномерно, есть некоторые искривление потока. Кривизна потока это наши напряженности электрического и магнитного поля. В зависимости от направления движения в протестантстве-времени(выбора системы отсчета), компоненты будут меняться, но суммарная "кривизна" потока не меняется.
Когда другая заряженная частица налетает на "чужой" поток заряда, она начинает бугурудить и сопротивляться этому произволу. В итоге пробная частица старается изменить движение так, чтобы минимизировать окружающую ее кривизну потока. Это проявляется как сила действующая со стороны поля на заряженную частицу.
Кванты.
В пространстве задается два поля электрон-позитронное и электромагнитные. Вместе они объединяются в единое поле через особые преобразования суть которые возможность описания эм поля как совокупности возмущения электрон-позитронов и наоборот. Далее через новое преобразование ведется квантование системы. Совсем упрощенно фотон превращается в пару электрон-позитрон, позитрон уничтожает старый электрон, а высранный электрон уже двигается по другому.
image.png278 Кб, 1256x706
181 481985
182 481986
>>1985
Твоя картинка не в тему, потому что гантель не сама себя тянет, а движется под влиянием среды (то есть нарушения закона сохранения импульса нет).
183 481987
>>1986
Архимедова сила?
184 481988
>>1987
Ее там нет.
185 481994
Смог бы бог поднять сингулярность?
186 481995
>>1983

> Получается движение без реактивных двигателей?


Ракеты и кометы - все сплошь на них летяють
187 481996
>>1995
Я неправильно выразился. Не движение без реактивных двигателей, а тяга без реактивных двигателей.
188 481997
ллллл
189 481999
>>1983

> шар с плотностью меньше чем плотность среды


> вселенная заполнена вакуумом


Сразу - вселенная заполнена вакуумом, и плотность среды равна нулю. Вся материя в космических объектах. Даже в растворах гетерофазные примеси практически не влияют на плотность, а ты говоришь о космосе, где на одну звезду приходятся десятки световых лет пустоты. Так что забудь про понятие плотности, и забудь о идее создать что-то более разреженное, чем вакуум.

Если же говорить о гравитации, то

> Но в вакууме есть звезды и планеты, так что если сделать шары астрономических размеров и вынуть из легкого шара все звезды и планеты, его плотность будет меньше средней плотности вещества во вселенной.


У нас есть звёзды на краю вселенной. Казалось бы, из за притяжения они должны приближаться к центру вселенной, но мы видим, что они от него отдаляются.

Наверное, получается, что любой пук в космосе будет намного эффективнее, чем надежда на гравитационное притяжение, которое убывает в квадратной зависимости от расстояния.

Да и вообще кажется, что даже песчинка оказывает на тело большее гравитационное воздействие, чем вся видимая вселенная. А если так, то аппарат пролетит до первой песчинки и станет её спутником.

И нельзя забывать про солнечный ветер - фотоны тоже умеют толкать объекты. Возможно то, что ты уберёшь половину вселенной наоборот будет толкать корабль назад из-за того, что воздействие света не уравнено.
190 482000
>>1994
Лее, маман в проходе упал, не можешь дойти до холодильника?
191 482001
>>1999

> Даже в растворах гетерофазные примеси


Ты химик что ли? Ну вот и иди в тред по химии, я реквестировал ответы физиков.

> краю вселенной


> центру вселенной


Ясн))
sage 192 482002
>>1983

>Поэтому он начинает убегать от легкого шара, но одновременно тянет его с собой. Получается движение без реактивных двигателей?


Привет третий закон Ньютона
Силы распиши, мамин изобретатель.
1485143784384.png9 Кб, 694x495
193 482003
>>2002

> Привет третий закон Ньютона


Где он здесь нарушается?

>Силы распиши


Пикрил. N1 и N2 в сумме дают 0, но F1 и F2 в сумме дают направленную вправо силу.
sage 194 482004
>>2003

>N1 и N2 в сумме дают 0


Ясно
195 482005
>>2004
Если движение установившееся, то иначе быть не может — иначе перемычка начала бы ускоряться.
sage 196 482006
>>2005
Чувак, у тебя среда оказывает давление на тяжелый шар сильнее чем на белый. Сорян конечно.
197 482008
>>2006

> среда оказывает давление


Что ты под этим подразумеваешь?
198 482009
>>2008
Ну я не знаю даже, может то, что среда тоже испытывает гравитационную силу и давит на тела? Типа рисуешь силу гравитации в сторону среды, будь добр и силы среды нарисуй.
images (1).jpeg8 Кб, 189x267
199 482013
Допустим, у меня есть задача приготовить сосиски. У меня есть две кастрюльки, одна уже, вторая шире, сделаны они из нержавейки. Сосиски начинают готовится, когда вода в кастрюле начинает вскипать. Вода должна покрывать сосиски. С одной стороны в узкую кастрюлю нужно добавить меньше воды, а с другой широкая кострюля имеет большую площадь соприкосновения с нагревательным элементом. Можно ли как-то рассчитать и как именно, где начнет быстрее кипеть вода, при условии, что все параметры, типа ширина/объем кастрюлей, размеры сосисок и т.д. нам известны, а так же температура нагревательного элемента одинакова в обоих случаях?
200 482014
если все автомобили будут ездить на водороде, как это повлияет на климат? вечный дождь?
мимоГрета
201 482018
>>2009
Силы гравитации нет, есть просто движение с ускорением, направленное в сторону большего искривления пространства-времени.
202 482019
>>1994

>поднять


Куда?
203 482022
>>1984

>В пространстве задается два поля электрон-позитронное и электромагнитные. Вместе они объединяются в единое поле через особые преобразования суть которые возможность описания эм поля как совокупности возмущения электрон-позитронов и наоборот. Далее через новое преобразование ведется квантование системы. Совсем упрощенно фотон превращается в пару электрон-позитрон, позитрон уничтожает старый электрон, а высранный электрон уже двигается по другому.


Спасибо, заодно стало понятно почему напряжение распространяется по проводам (по тому, где много свободных жлектронов) а не равномерно во все стороны от источника.
204 482028
>>2018

> Силы гравитации нет, есть просто движение с ускорением, направленное в сторону большего искривления пространства-времени.


Это не я >>1983 писал.
205 482031
>>2013
Можно. Нужны экспериментальные данные
206 482032
>>2014

>вечный дождь?


резкий и вечный парниковый эффект
не ну а откуда ещё брать миллионы тонн водорода, кроме как энергозатратным разложением воды, используя классический уголь\нефть

но если забыть про этот досадный факт - то ничего не будет. Я посчитал на калькуляторе - машина на водороде производила бы в день воды не больше, чем выдыхает за день человек
207 482033
>>2028
а хули на вопрос не отвечаешь, петя?
208 482034
>>2033
Иди нахуй, дебил.
209 482035
>>2034

>пок


петя, тебе помочь пытаются, а ты клюёшься
210 482036
>>2035

>обижается


>кукарекает


Ясно.
1477605461648.png9 Кб, 694x495
211 482037
>>2033
>>2009
Со стороны среды на шары силы нарисованы, а со стороны шаров на бесконечную среду рисовать незачем.

Алсо, есть максимально упрощенный вариант самоходного вселенского корабля: невесомая сфера, пустая внутри и с закрепленной на ней точечной массой, находящаяся в однородной среде.
212 482038
>>2032

>откуда ещё брать миллионы тонн водорода, кроме как энергозатратным разложением воды


природный газ
213 482039
>>2019
В сторону доменной стенки.
214 482040
>>2000
Че ты несёшь, поехавший?
215 482082
Берём генератор постоянного тока, ставим его на изолятор. Пускаем от него один провод на нагрузку, со второго контакта нагрузки пускаем провод к хорошему заземлению.
Это будет работать? Откуда будут браться электроны в генераторе, взамен убегающих к нагрузке и потом в землю?
Или это заработает только если генератор будет заземлён.
216 482083
>>2082
Нет, не будет.
Излишний/недостачющий заряд останется на изоляторе, поскольку он ЭЛЕКТРОЕМКОСТЬ.
217 482089
>>2037
Вощето звезды друг с другом гравитационно не связаны. Твой пузырик скорее будут гонять по вселенной эфирные ветра чем появится тяга от гравитации соседних звезд.
218 482091
Вопрос по разностным численным методам решения задачи Коши и краевой задачи. Я пока только знаю что есть именные методы решения дифф. уравнений классической математической физики, и есть, например, метод конечных разностей(МНР). Правильно ли я понимаю что для типовых именных методов уже доказаны сходимость и устойчивость, теми кто их и разработал, а МНР это как бы конструктор решения и требует для каждой конкретной задачи подбирать разностную схему для которой удастся доказать сходимость и устойчивость ?
219 482092
>>2091
Вряд ли тебе здесь ответят, попробуй в /math/
220 482094
>>2038
погуглил, спасибо

Тогда только вторая часть поста
221 482098
>>1882
Нихрена не понел правильно ли я тебя понял. Поэтому расскажу что енто такое корпускулярно волновой дуализм.

Корпускулярно волновой дуализм состоит в том что любая частица тянет за собой волну-пелота. Усё. Но если говорить конкретно про фотон, то это составная частица из многих более мелких частиц и волновые свойства у фотона выражены именно периодичной структурой расположения этих частиц. Ну и конечно же помимо этого у фотона есть пелотная волна.
222 482099
>>2098

>фотон, то это составная частица из многих более мелких частиц


чево блят
223 482100
>>2089

> Вощето звезды друг с другом гравитационно не связаны.


В галактиках связаны же.

Подвох скорее всего состоит в том, что бесконечную во все стороны среду некорректно рассматривать почему-то. Ну тип, если поместить эту конструкцию на расстоянии полрадиуса от центра шара из окружающей среды и рассматривать средовые шары с увеличивающимся радиусом, то можно добиться чтобы со стороны среды гравитационная сила действовала куда угодно (но всегда к центру средового шара).
225 482102
>>2101
ты че, ебанутый, блядь? пошел нахуй из треда
226 482103
>>2101
Охуенно, спасибо, прочитал полностью
227 482104
>>1846

>Правильно я понимаю, что если поставить фонарик на тележку и пустить ее ехать по направлению к детектору, то свет достигнет детектора именно со скоростью света, то есть скорость тележки не будет с ней суммироваться?


Нет не верно. Фотон не может двигаться с цэ относительно любой системы отсчета. Это противоречит логике. Фотон движется с цэ относительно некоторой системы отсчета, связанной со средой от которой фотон отталкивается в процессе своего движения. Эта система отсчета может уже двигаться относительно фонарика, тележки и детектора.

>А если тележка едет в обратном направлении с произвольной скоростью, будет ли скорость тележки вычитаться из скорости света?


Да. Скорости складываются и вычитаются обычным образом.

>А если допустим, два фотона в момент времени t=0 начали удаляться друг от друга в противоположном направлении - значит ли это что в системе отсчета на одном фотоне, второй будет удаляться от него со скоростью 2с?


Верно.
228 482105
>>2104
Шизоид эфирный, прочь из треда!
229 482119
>>2037
Ты не нарисовал все силы. Всё, ты меня утомил уже. Объяснять что-то на дваче просто раздражает. Твои ебучие шары, будут испытывать на себе силы от среды, так как они эту среду к себе тянут, а она (жидкая падла) будет на них давить. И эти силы давления, полностью уравновешиваются теми силами, которые возникают из-за притяжения шариков к среде.
230 482126
Допустим, у меня есть оружие, которое может отключить мозг без явного физического повреждения (т.е. кусок арматуры не подойдёт). Как бы работало это оружие? Возможно ли оно вообще? Чтобы типа как эми перегружало мозг как-нибудь, чтобы он просто выключался.
1517055193501.jpg614 Кб, 1422x1415
231 482137
>>2126

>Как бы работало это оружие? Возможно ли оно вообще?


Ты аниме придумал.
232 482141
Можно ли устроить бои лимфоцитов разных людей/животных, чтоб узнать, чей иммунитет сильнее? Или нейтральные для всех внешние условия никак не устроить?
233 482142
Если взять отдельно по 100г каждого белка, вырабатываемого в теле человека, то какими они будут на вкус-цвет-ощупь в большинстве своём?
234 482144
>>2142
Ебанутый? Разные белки различны по свойствам.
235 482145
Вопрос по физике. Слыхал я, что согласно то ли ОТО то ли СТО, то ли тому и другому, существует пространство-время, что гравитация искривляет пространство-время.
Всё это умозрительно как-то сложно выглядит. Может всё несколько проще?
Попробую на примерах:
1. Гравитационное линзирование - обыкновенная интерференция света, с учётом его движения по орбите небесного тела, вызванного гравитационным притяжением, не искривлением пространства.
2. Скорость "времени" у горизонта событий сингулярности. Поговаривают, что время сильно замедляется. Время же суть синтетическая величина, обозначающая скорость течения процессов. А раз так - получается гравитационное взаимодействие снижает эту скорость, выступая в качестве некого обратного вектора силы для прочих элементарных взаимодействий.

Я хуй без образования, потому не знаю, на сколько это противоречит имеющимся сейчас фактам, в виде прочих экспериментов? Мне было бы гораздо более понятное существование этого мира с пространством, которое жёстко, статично, существует вне зависимости от материи и её наличия.
236 482146
>>2141
Оно только в системе работает (лейкоциты, антитела, лимфоузлы и тд). Сферический лейкоцит в вакууме никого сам не обнаружит без антител еще и помрет сразу.
237 482147
>>2145

>которое жёстко, статично, существует вне зависимости от материи и её наличия.


Кто тебе мешает так считать в рамках планеты, на которой ты живешь? Теории это больше разминка для ума ученых разного разлива и построение теоретической модели для будущих потребностей.
238 482149
>>2147
В рамках планеты и своей маленькой жизни я так и живу, но не хотелось бы быть обманутым самим собой, это просто печально.
Это всё очень сильно тормозит восприятие мной информации из более серьёзных источников, чем научпоп. Каждый раз, когда я пытаюсь что-то почерпнуть из мира высокой физики, что с точки зрения огромных масштабов, что с точки зрения квантовых взаимодействий, я испытываю серьёзное отторжение в плане некой естественности. По квантовым вопросам есть тоже мысли о том, что кто-то слишком сильно усложнил вопрос, всё должно быть чуточку проще, естественнее.
Так, например, я очень долго одуплял что такое спин, вернее как он "возможен" в рамках реальности, а не бумаги, читая различные определения на русском, пока не прочёл развернутое определение-разжовывание на английском, где оказалось речь идёт чуть о другом, нежели о том, что я старался представить. Пусть это и чуть другое, но для примера того, что и зачем я спрашиваю, пойдёт.
239 482150
>>2119

> она (жидкая падла)


Она не жидкая и не давит на них.
240 482152
>>2150
Ну и в чём тогда прикол?
241 482156
>>2149
Более высокая физика, это просто более абстрактная математика. Вообще в физике нет универсальной теории, разберись с классикой, потом кидайся на другие.
Впрочем тем дальше в лес, тем более абстрактные вещи возникают. Хотя теорию групп вполне подъемная даже для школьников.
С квантами есть вполне годный визуальный способ перейти от классике к квантам через геометрическое квантование.
242 482162
>>2149
Время не замедляется в твоей локальной системе никогда, оно замедляется только по отношению к другим системам.

Когда говорят что время у чёрной дыры замедляется время, то не у того, кто на неё падает. У него всё процессы идут как обычно.
243 482165
>>2162
Т.е вообще нет смысла говорить о замедлении времени в твоей системе в абсоюлюте / без ссылки по отношению к чему?
Это как и с движением: покой и движение это одно и тоже. Относительность движения, см. Коперник / Ньютон.
>>2149
Ты и есть локальная система, ты не можешь понять как двигаются процессы в твоей системе, потому что ты и есть эти процессы, и то чем ты измеряешь, это и есть эти процессы. "Скорость" течения времени есть только по отношению к чему то, а не в абсолюте.
244 482166
>>2145
Нормальные физики когда читают отоблядские басни, то переводят пространство-время как эфир, гравитационное линзирование как обычное линзирование на среде вакуума, искажение времени как искажение скоростей процессов при нормальном времени. Потому что нужно мыслить нормальными моделями, а не херней не имеющей физического смысла.
245 482167
>>2166
пошол нахуй пидорас в жопу ебущийся
тебе здесб не рады
246 482168
>>2166
говна наверни скот эфирный, чтобы твою мамку негры выебли
247 482169
>>2102
>>2105
>>2167
>>2168
Не хуево так бомбит кому-то. Хех, мда.
248 482170
>>2169
а хули ты распространяешь ложь, мм сука? такие как ты опасны.
249 482171
>>2170
Где ложь? Покажи.
250 482172
>>2171
да ты еще и тролль. все ясно.
251 482181
В нашей изогнутой вселенной сумма углов в треугольнике все-таки равна 180 градусов или хотя бы на 10-100 градусов меньше/больше?
252 482190
>>2181

>изогнутой


Плоской же. Насколько можно измерить.

>180


Насколько можно измерить-ровно.

Точную точность не помню, но если кривизна и есть, то размер вселенной, т.е. если она таки изогнута и замыкается сама на себя, а не плоская, вроде больше как минимум 1.8 трлна световых лет, а не как у видимой 13.8 млрд.
253 482191
>>2181
Хотя вот интересно, ровность суммы углов измеряют на больших маштабах, где рябь искривлений от скоплений масс не влияет, а что если измерять например на земле? Или тут, на орбите солнца. Интересно, что больше влияет.
15714418028530.webm10,3 Мб, webm,
626x360, 1:35
254 482192
зачем вороне ложка если у нее нет рук
255 482199
Посоветуйте хардкорных книжек по квантовой теории, после освоение сивухина. Желательно с математическом дополнением.
256 482200
257 482201
>>2200
Спасибо
5833poster.jpg46 Кб, 450x250
258 482214
>>1090 (OP)
Если нагреть тело до такой температуры, что его излучение уйдет за видимый спектр, то как оно будет выглядеть и перестанет ли оно светиться?
259 482217
>>2214

>его излучение уйдет за видимый спектр


>как оно будет выглядеть


Как хуй Ашота.
260 482221
>>2217
Никогда не видел. Покажешь?
261 482223
>>2214
нет не перестанет
нагретое тело излучает во всем спектре, в том числе в видимой части
вопрос только в интенсивности
262 482233
Почему супертехнологичные атомные реакторы до сих пор греют воду для электричества, а не применяют какие-нибудь хитровыебанные квантовые штучки? Учёные слишком тупые что-нибудь новее паровой машины придумать?
263 482237
>>2233
Потому что это никому нахуй не нужно.
264 482239
>>2237
Повышать кпд не нужно? Он что, уже 100%?
265 482240
>>2239
Предложи безопасную технологию для увеличения КПД и её с радостью примут.
266 482242
>>2240
1. мне это не надо
2. Я не умный учёный, кембриджов не кончал
Неужели прям ничего другого, никакого способа конвертации, кроме как через посредство допотопной пароваркой, нету?
267 482245
>>2242
А зачем? Просто чтобы конкретно тебе было хорошо от сознания, что они работают на "умных штуках"?
268 482246
>>1090 (OP)
Где вы читаете новости прогресса?
269 482247
>>2242
Пароварка норм конвертирует. Да, КПД не 100%, но там теоретический максимум не 100% как бы, при таких температурах холодильника и нагревателя (тепловая машина же, епта).
image.png360 Кб, 604x408
270 482248
Если возраст Вселенной 14 миллиардов лет, то как она может иметь диаметр в 92 световых года?
271 482249
>>2248
Если ты ходишь всего несколько метров в секунду, то как ты умудряется подняться по эскалатору на 90 метров за 30 секунд?
272 482250
>>2249
И где во Вселенной эскалатор?
Почему Вселенная расширялась быстрее предельной скорости?
273 482253
>>2248
где ты эту чушь прочитал? имею в виду вторую часть предложения
274 482254
Расскажите где в Германии можно заниматься плазмой. Хочу вкатиться в Европу через DAAD, заниматься там ЭРД или хотя бы источниками плазмы.
275 482257
>>2250
Предельная она только для движения по пространству. А вот само пространство не ограничено этим.
276 482259
>>2257

>А вот само пространство не ограничено этим


Пространство тоже форма материи.
277 482260
>>2259
Сам придумал? Или где-то прочитал?
278 482261
>>2250

>И где во Вселенной эскалатор?


У него хреновый пример, ходить по эластичной ленте, которую растягивают.
279 482262
>>2260
Если пространство имеет конечную энергию, то оно является материей.
>>2261
Кто растягивает? И почему его растягивание нарушает теорию относительности?
helmet.jpg91 Кб, 800x999
280 482263
Почему нельзя хранить антипротоны в виде разреженного холоднго газа, ведь в силу отрицательного заряда они не смогут пройти через электронную оболочку атомов к ядру, не облядая достаточно большой энергией?
281 482264
>>1090 (OP)
Вопрос немного туповат, но всё-же, задам его здесь...
КУДА ВПАДАЕТ ВОЛГА?
282 482265
>>2264
Издревле было известно, что через волго-донские каналы, река Волга соединяется с рекой Дон,
и впадает через Азовское, Чёрное, и Средиземное море - в Чермное море (ныне именуемое как - Красное море).
Однако если пойти ещё дальше, то через гибралтарский пролив, она впадает ещё и во все океаны, нахой,
а затем, испаряясь - впадает ещё и в атмосферу, и во все облака, в том числе и в облака в Антарктиде.
Так, что, таким образом, Волга эта, впадает ещё и в Исток реки Иордан. Лол.
image.png77 Кб, 528x733
283 482266
Здесь написано, что линии напряженности начинаются/заканчиваются на заряде и уходят на бесконечность. Получается, что заряд распостраняет напряженность бесконечно далеко, и теоретически мы можем сверхточным прибором зафиксировать напряженность которую образует, допустим, какой -нибудь электрон на расстоянии в овердохуя км от него? Или я что-то неправильно понял?
284 482268
>>2262
Тёмная энергия.
Не нарушает.
285 482269
>>2268
ты скозал?
286 482274
>>2266
Да, все так. ЭМ взаимодействие распространяется на бесконечность. Два заряда будут чувствовать друг друга на миллиарды световых лет от друг друга. Главное чтоб эм волна успела долететь.
Впрочем ситуация немного другая, если вселенная имеет хитрую топологию, вроде взаимодействие происходит на 4-мерных аналогов тора или ленте Мебиуса, но тебе это знать не надо.
287 482276
>>2266
напряженность электрического поля обратно пропорциональна квадрату расстояния, а это сравнительно быстро затухающая функция
математически это означает что поле исчезает только на бесконечности, но это только теория. На практике - нет, поле исчезнет гораздо раньше и заряды чувствовать друг друга не будут
288 482277
>>2269
Пресловутый космологический член ввелся еще задолго до открытия темной энергии. Так что действие темной энергии давно включено в теорию.
289 482278
>>1090 (OP)
За ужином в кругу ученых попался типаж алкоголика-балагура, который что-то знает о своем деле в сфере естественных наук, но по культурным взглядам застрял в 1953 году.

Публика неловко слушала его бесконечные анекдоты из прошлого века, истории о том, как он работал в Канаде, где женщины благодарили его за защиту от феминисток, и комментарии о том, какие сейчас тупые студенты.

Тут я предложил ему выпить за гендерное равенство, многие начали смеяться, и понеслось.
290 482279
>>2278
че сказать-то хотел?
1306948651087.jpg373 Кб, 1000x1000
291 482290
>>2263
Бамп вопросу.
292 482291
Если вдруг на большом адронном коллайдере обнаружат гравитон, будет ли это означать, что гравитация квантуется, как и остальные 3 типа взаимодействий? Поможет ли это разработать теорию квантовой гравитации?
293 482292
>>2291
И так считается, что гравитация должна как-то квантоваться. Другое дело каким именно это происходит. Теорий напилили уже на данный момент очень дохуя, осталось их только проверить и выбрать нужную. А с этим проблемы. Гравитация из слабейших взаимодействий, если мы хотим непосредственно измерить гравитационное взаимодействие двух объектов, надо как-то исключить другие. На БАК и прочих коллайдерах мы только нащупываем мощные поля, т.е по сути создает ворох говна, вместо исключения его.
Впрочем тут идут другим путем, нам в лаборатории нужен релятивистский объект: ЧД или холодная нейтронная звезда, желательно несколько штук. Сам понимаешь невозможность такого на данным момент.
294 482298
>>2292
вот кстати про черную дыру. Где-то читал что на БАКе есть риск того, что возникнет крохотная черная дыра и зохавает всю планету. Но ведь это ересь ебаная, потому что противоречит закону сохранения энергии? Конечно, энергия пучков в коллайдере велика, но она отнюдь не такая огромная, что смогла бы всосать все вокруг в себя.
295 482300
>>2298

> противоречит закону сохранения энергии


не противоречит, потому что излучение хокинга
296 482302
>>2300
что - излучение хокинга? его результат как-то отменит все движение массы Земли в черную дыру (гипотетическую)? ты щас говоришь про начальное и конечное состояние, а я про траекторию вдоль профиля энергии. не может ведь пучок пидорских частиц привести к огромному потенциалу энергии для поглощения планеты
297 482303
>>2302
если он будет поглощать материю и распылять её в виде квантов, там энергии дохуищща может проявиться (но ты мои слова на веру не бери, я не настоящий сварщик, гуманитарий я, можешь начинать смеяться)
298 482304
>>2303
это рофел или что?
299 482305
>>2304
есть что возразить?
300 482306
>>2305
да. закон сохранения энергии.
301 482308
>>2306
И каким образом он возражает чёрной дыре, стягивающей на себя одеяло всей нашей галактики, например?
302 482309
>>2308
а о ней речь не шла
303 482310
>>2309
ну тогда поясняй за гипотетическую
304 482311
>>2310
еще раз, с самого начала
есть чья-то гипотеза, что в результате экспериментов на БАКе может возникнуть мелкопиздюшная черная дыра и тогда она засосет в себя все вокруг. То есть эксперимент приведет к созданию мощного гравитационного поля, энергия которого приведет к сжатию всей материи Земли в спичечный коробок. Я спрашиваю, как это может произойти, ведь по закону сохранения энергии для рождения такой черной дыры нужно затратить энергию. Которой, очевидно, нет на БАКе.
305 482312
>>2311

> мощного гравитационного поля


сначала не мощного, но по мере пожирания окружающего пространства, умощняющегося

> для рождения такой черной дыры нужно затратить энергию. Которой, очевидно, нет на БАКе.


тебе очевидно, а мне не совсем
306 482313
>>2312

>сначала не мощного, но по мере пожирания


то есть энергия будет увеличивать саму себя? ебонат штоле?

>тебе очевидно, а мне не совсем


последний эксперименты на коллайдере проходили на энергии столкновения частиц около десятка тэраэлектронвольт. Это 10^-7 Джоуля. Порядок величины тебе понятен?
307 482314
>>2313

>10^-7


10^-6
быстрофикс
308 482316
>>2313

>то есть энергия будет увеличивать саму себя? ебонат штоле?


лесной пожар представляешь как работает?
>>2314
вопрос в том, могут ли устойчивые чёрные дыры возникать на подобных масштабах (если концентрация массы будет слишком большой для её объёма, не приведёт ли это к цепной реакции типа чд)
Электромагнитный ракетный двигатель 309 482318
Анон, а реально ли сделать такой реактивный двигатель в котором полностью либо частично функцию стенок сопла будет выполнять электромагнитное поле? Типо магнитная ловушка, которая помогает корпусу ракетного двигателя сдерживать давление.
310 482320
Ну или как нибудь совместить сгорание рабочего тела и его "выталкивание" потоком какой нибудь хуйни-малафьи. Типо рельсотрона, который "плюётся" сгорающим жидким ракетным топливом.
311 482322
>>2233
Реактор сам по себе производит тепло и радиацию (которая тоже в тепло уходит). Выгодней всего тепло переводить в электричество посредством паровой машины, пусть и в виде каскада турбин. Как в нф, напрямую энергию снимать, еще долго не получится. Гораздо выгодней повышать кпд, снова пережигая топливо в реакторах следующего поколения, оно сейчас очень мало расходуется, 90 процентов топлива уходит в отходы.
312 482323
>>2318
СПД?
313 482326
>>2323
Совместить ионный и ракетный двигатель. Сгорание и выталкивание электромагнетизмом.
314 482327
>>2323
Типо сначала пропускать через VASIMR топливо, и потом его еще и пождигать
315 482338
>>2318
Реально, и уже делают.
Только все упирается в одно - компактный и мощный источник электроэнергии. А если хочешь йоба сверхпроводящих магнитов, тогда тебе нужна система охлаждения, тоже компактная. А иначе выйдет залупа весом десяток тонн и тягой в один пук.
316 482395
Каким был климат России во времена Петра I?
baka2.jpg9 Кб, 163x200
317 482433
>>2263
Ну же, никто не знает?
318 482434
>>2395
У шведов спроси.
319 482573

>ТРЕД ТУПЫХ ВОПРОСОВ


Почему в этом треде и в /sci/ никто не постит?
sage 320 482580
>>2573
Потому что обо/сцай - параша для говна, шизиков и быдла (тебя)
321 482582
>>2580
Ты с /zog/ перепутал, гной. Хотя я и там сижу.
sage 322 482584
>>2582

>Хотя я и там сижу.


То, что ты червь-пидор давно не новость.
323 482592
>>2584
Это был не вопрос, а значит отвечать было не обязательно.
sage 324 482593
>>2592

>эти униженные мольбы


Самый настоящий.
325 482604
>>2263
У электрона отрицательный заряд.
326 482605
>>2573
На дваче остались слабые хомячки. Появляются, попадают под набеги >>2580-быдла и убегают.
327 482616
>>2604
Как и у антипротона. Перечитай мой пост внимательно.
328 482617
Какой вариант интерференции у фотона в действительности?
а) Если фотон волна, то у него интерференция примерно как у кругов на воде. То есть фотоны в противофазе гасят друг-друга, а фотоны в фазе усиливают друг друга.
б) Если фотон мелкая частица, окруженная волной-пилотом, то интерференция пилотной волны будет отклонять траекторию частицы.
329 482618
>>2617

>в действительности?


С чего бы начать... дело в том, что мы не можем узнать, оно и не важно. Оба твоих варианта для нас будут одинаково выглядеть. Так не похуй ли?
330 482621
>>2618
Нет не похуй, различия все равно должны быть.
331 482622
>>2617
Как там в 1910х? Уже известно, что фотон - ни то, ни другое.
332 482624
>>2622
Значит второй вариант верен.
333 482625
>>2616
У электрона отрицательный заряд. У протона положительный заряд. У нейтрона заряд - от названия.
Ещё раз пизданёшь антинаучную ахинею - внимательно пиздану учебниками физики и химии.
334 482626
>>2621
Почитай кванты и квантовую электродинамику, и вернемся с тобой к этому вопросу.
335 482627
>>2263
Они друг от друга отталкиваются. Как ты их в куче то хранить собрался? Если бы ты говорил про атомы антигелия или молекулы антиводорода, другое дело.
336 482628
>>2627
Мне кажется он не поймёт, в таких случаях надо прямо в лицо говорить максимально.
337 482646
>>2625
ты долбоеб ебаный и не знаешь что такое антивещество в целом и антипротон в частности
прежде чем хуями крыть, хоть сам разберись в вопросе

>>2263
хуй знает, во-первых антивещество хуй просто так получишь
даже если их получить, возможно, какое-то время они будут стабильны как раз из-за отталкивания с электронами окружающего вещества. вопрос в том, какое время
338 482648
>>2627

>Как ты их в куче то хранить собрался?


Как любой сжатый газ.
339 482649
>>2625

>У протона положительный заряд


И какое это имеет отношение к моему вопросу?
340 482654
>>2648
Круто. Начнет электроны выбивать из того, чем сжимать будешь.
341 482695
>>2593

>эти униженные мольбы


Проиграл почему-то.
342 482703
>>1090 (OP)
Пусть вероятность выиграть в игру - составляет 55%, а проиграть - 45%.
Какова вероятность n выигрышей подряд, и n проигрышей подряд?
343 482707
>>2654
А если взять диэлектрик?
344 482708
>>2263
А как хранить протоны в виде того же газа?
345 482710
>>2708
В герметичной емкости.
346 482712
>>2710
Ну, и храни свои антипротоны, в герметичной антиемкости в антиатмосфере антиЗемли.
347 482729
Тёмная материя.

Почему никто не рассматривал идею о том, что ОТО нужно подбравить, чтобы она работала норм.

Например, мы же знаем, что события в ОТО не одновременны, и в одной системе наблюдателя события могут происходить позде или раньше. А что если на большом удалении (галактики) возникает некий эффект искривления пространства времени таким образом, чтобы попадать под эффекты "тёмной материи". На маленьких расстояниях этот эффект не заметен, но на больших гравитация кажется выше чем в реальности. То есть со стороны андромеда: это спиральная галактика, которая врашается как говорит дополнительная масса. А на маленьких расстояниях эффект не проявляется и мы не имеем массы эффекта.

Поняли?
348 482730
>>2729
Это происходит из за большого искажения пространства времени для наблюдателя. Галактика андромеда находится в гравитационном колодце и всё искажется так, что для стороннего удалённого наблюдателя появляется больше массы, а для наблюдателя внутри андромеды массы меньше.

Ну.. как со временем, типо события не одновременны, если разность скоростей близка к скорости света.
И гравитационные эффекты не одинаковы, если масса объекта очень высока.
349 482735
>>2729
Хуй его знает, иди где-нибудь почитай еще раз чего-нибудь умного по темную материю, особенно почему ее нельзя списать на сторонние эффекты гравитации.
Да и зачем придумывать, если уже сейчас научились "видеть" темную материю(на самом деле видят разницу между плотностью темной материи и светящейся) и она выглядит именно как материя или вещество.
350 482744
>>2735
Нашли эфир?
351 482745
>>2744
Нет, он же неуловимый джо.
352 482752
>>2744
>>2745
давно нашли (просто никому не нужно чтоб вы дома шатали да магнитосферу размагничивали)
353 482755
Почему гравитация обратно пропорционально квадрату расстояния между телами? То есть я понимаю, что это всё можно вывести формулами, но хочется интуитивного объяснения. Например, я интуитивно понимаю, почему количество попадаемого света на тело подчиняется этому закону, но почему гравитация?
354 482756
>>2755
То есть требуется такое объяснение, чтобы его можно было дать пиздюку 8-икласснику, чтобы тот понял, почему формула такая, а не принял на веру.
355 482757
>>2756
Такого объяснения не существует.
356 482758
>>2757
Лишь потому, что никто на самом деле не знает почему. Подобрали мат.модель, которая вроде подходит под эксперимент и в рот её чих-пых.
357 482763
>>2757
>>2758
Да ладно? Некоторые ютуб-хуи умудряются относительно наглядно показать релятивизм, неужели куда более баянистая формула не имеет не обязательно точного, но примерного объяснения?
358 482764
>>2757
Так какое тогда есть хотя бы наиболее понятное обяснение, с минимумом выкладок, чтобы на уровне "хуе-мое, имеет смысл"?
359 482765
>>2764
Не обязательно для 8-иклассника, но чтобы что-то интуитивно понятно было.
361 482767
>>2766
Так её ж хуями покрывают. То есть даже как аппроксимация, эта теория лажает, нет?
362 482768
>>2755
Вообще это следствие обобщения теоремы гаусса. Обратный квадрат появляется из 3-мерности (тут важно как считать интеграл, по 4-объем вообще нуль выйдет непрерывности.) области, которое порождает поле. Вообще это верно для любых скалярных полей со симметриями. Гравитация в приближении(в стационарном случае или когда относительные скорости значительно ниже световой) действует как раз как скалярное поле.
kTd8rfZEbhg.jpg198 Кб, 1165x770
363 482770
>>2767

>Так её ж хуями покрывают.


настоящего учёного это ебать не должно. почитай, разберись.
364 482771
>>2770
Тогда почему бы не реанимировать флогистон?
365 482772
>>2768
Как это всё перефразировать, чтобы пиздюк понял?
366 482774
>>2771
для чего? если считаешь, что имеет смысл, попробуй.
367 482776
>>2770
Я могу разобрать с пиздюком все 1488 различных противоречащих друг другу теорий, но я считаю что в текущей ситуации будет продуктивнее дать обяснение согласующееся с наиболее проверенными из них.
368 482777
>>2776
тогда просто объясни, что в плоскости, в которой находятся центры этих объектов, площадь выражается квадратичной величиной а окружность линейной
369 482781
Почитал тредик. Подумал, что мне надоело баловаться научпопом, потому что он не формирует полной картины. По каким книженциям или лекциям лучше учить физику и откуда решать задачи? Я физику закинул классе на десятом, до этого олимпиадки городские в своем Мухосранске выигрывал, но потом пошёл программировать за 300к/наносек. Сейчас возникло непреодолимое желание восполнить проёбанное.
370 482787
>>2781
Маст хэв - матан, линал, ТФКП. Ты должен знать и уметь работать с дифференциалами, специальными функциями, матрицами, линейными операторами и тензорами. Ну еще не помешает статистику и теорвер, но они обычно включаются в курс физики.
Если втащишь, то еще накати следующее, однако это излишне, но чем раньше познакомишься тем лучше. Тензорный анализ, диффгеометрия, диффуры, теория групп.

Физика.
Фейнманские лекции
Берклиевский курс физики
Савельев
Сивухин
Не плохой Griffiths D. Introduction to... но он на пендосском, без русика.
371 482788
>>2781
Тебе зачем? Просто мировоззрение сформировать? Тут хватит аудио/видео лекций, вики, ресурсов со статейками, и самоорганизации. Хочешь чего-то большего, систематизации и собственных потуг - очка/заочка/вечерка/наставник и т.д. То есть без помощи зала тяжеловато будет.

>>2787
Анон за физику спрашивал, а не матан. Когда понадобится матмодель, вас вызовут. До тех пор продолжайте добавлять мерности к тензорам двухсотмерных бран, ваши расчеты очень важны для нас, оставайтесь на линии.
372 482789
>>2787
>>2788
Благодарю, аноны
373 482791
То что физики измерили период распада изотопа ксенона-124, означает что эта частица появилась раньше чем возникла наша вселенная?
374 482792
>>2791
Нет, же.
Каждый радиоактивный изотоп за определенный интервал времени имеет определенную вероятность распада. То, что атом имеет такой большой период полураспада просто означает, что вероятность распада очень низкая. Просто физики несколько раз зарегистрировали редкое событие и подсчитали его статистику и через него вывели период полураспада.
375 482801
Я читал, что гравитация не является силой, а всё дело в искривлении пространства. Но почему в этом искривленном пространстве из состояния покоя тело начинает ускоряться в сторону большего искривления?
376 482804
"Наука это идеология толкования фактов?"
vh6URD7uxLo.jpg112 Кб, 908x474
377 482806
>>2804
"Наука это вера в невежество экспертов!"
7166140.jpg89 Кб, 440x274
378 482807
>>2755

>Почему гравитация обратно пропорционально квадрату расстояния между телами?


>>2763

>объяснения


Геометрическое. Квадратичная зависимость имеет такую же форму, какак и искривление пространства-времени от массивного объекта.
379 482808
>>2801

>Но почему в этом искривленном пространстве из состояния покоя тело начинает ускоряться в сторону большего искривления?


Чтобы продолжать двигаться по прямой. Т.к. чтобы двигаться не по прямой к телу/телу придётся приложить усилие.
011.png30 Кб, 554x192
380 482820
>>1090 (OP)
окей. Кто из вас больных ублюдков науковеров это был?
381 482824
>>2820
Виктор Вахштайн
382 482827
>>2808
А почему оно даже с нулевым ускорением движется по прямой, вместо того, чтобы стоять на месте? Как вектор скорости, полностью направленный в координату времени, влияет на движение в пространстве?
383 482828
>>2820
Будто на трапов, BDSM и sissy только науковеры дрочат. Каков мудак
384 482829
>>2827
Потому что везде криво, ты никогда не стоишь на месте, а всегда движешься, и если не прикладывается усилий чтобы по кривой, то движешься по прямой.
385 482830
>>2829
Ну если я правильно понял, то ты движешься во времени, а вектор движения в пространстве равен нулю. Почему движение во времени вызывает движение по кривизне пространства?
386 482831
>>2830
1)Это едино.
2)Ты всегда движешься в пространстве, лалка, и всегда действует куча услилий, препятсвующему твоему движению по прямой в место наибольшего искривления:сила реакции опоры земной поверхности на которой ты стоишь, центростремительная сила когда вращаешься на земле вокруг солнца, она же, когда в солнечной системе движешься вокруг центра галактики, она же, когда летишь в галактике к чему-то по касательной/вращаешься вокруг чего-то с ней и т.д.
387 482832
Когда говорят, что во время аннигиляции 1 грамма материи и 1 грамма антиматерии выделяется ~1.8×1014 джоулей энергии, подразумевается, что в одном грамме материи (и антиматерии) содержится ровно половина этой энергии, а во время аннигиляции они (по аддитивной логике) дают вот эту цифру?
388 482833
>>2831
Есть координаты пространства, есть координата времени, вместе они формируют единое четырёхмерное пространство-время, так? Так вот, у нас скорость нулевая в трёх координатах пространства, а в координате времени скорость C. Но почему кривизна пространства начинает менять координаты скорости?
389 482835
>>2833
Можно ответить что-то, но вижу тебе смысла нет, это всё абстракции, а тебе нужно узнать как оно на самом деле, а такого тебе никто не ответит, кроме сугубо их шизодиндных фантазий, если они считают что всё точно, 100% именно так и есть в реале.
390 482837
>>2832
А ты посчитай через E=mc2
391 482839
>>2833
Держи, если переваришь, вроде несложно:
https://physics.stackexchange.com/questions/102910/why-would-spacetime-curvature-cause-gravity
392 482840
>>2837
Для одного грамма получается почему-то 9 x 1014. ЯННП.
393 482841
>>2840
Вернее, 89875517873682 джоулей - это, получается, 8,9x1013, что ли. Я уже вообще ничего не помню.
Просто почему спрашиваю, пытаюсь вычислить, сколько нужно антиматерии, чтобы взорвать Землю, и в интернете наткнулся на вот этот калькулятор:
https://www.edwardmuller.com/index.php?Page=calculator
А там какие-то цифры дурацкие: получается по его таблице, что нужно где-то 800 млн. мегатонн, что абсурдно, потому что это около восьми Чиксулубских астеройдов.
Другие источники указывают вообще ебанутые цифры - вроде, массы Эвереста.
394 482846
>>2835
Ну я не хочу понять истину бытия, я хочу понять, почему у нас увеличивается скорость в рамках существующих теорий. Почему мы не можем перемещаться сквозь время, сидя на месте?
>>2839
В ангельский (особенно технический) плохо могу
395 482853
>>2781
Если хардкорный аутист то можешь попробовать
Thorne Blandford - Modern classical physics
если попроще то
Halliday & Resnick - fundamentals of physics

для задачек еще есть Walter Greiner
еще думаю можно motionmountain полистать
совецкий дрист - ненужен

А лучше всего лекции на ютубе смотри, с книжками только заебешься и бросишь.
396 482862
>>2853
Спасибо

>совецкий дрист - ненужен


А почему?
397 482870
>>2862
Из-за давление мехматовской школы.
Короче в совке физики обучались математиками. Хоть математика в совке была одна из лучших в мире, но чистые математики не могут в физику, совсем. Из-за этой доминации физики занимались совсем не тем, а потом писали книжки соответствующего качества. В итоге у нас по физики нет ничего достойного на уровне общей физики, да и годного учебного пособия по классической теории поля нет.
art-bin-yang-5501668.jpeg225 Кб, 811x1178
398 482873
Что делать, если результаты, опубликованные в научной статье (точнее в двух статьях) не воспроизводятся? Более того, по моим прикидкам, обозреваемый подход вообще не должен работать, или будет работать только на вычислительном кластере / суперкомпьютере. Есть статья (даже две) с анализом криптографических функций. Из-за текущего состояния дел статьи представляют скорее теоретический интерес. Но дело в другом. Я реализовал алгоритм, как было указано, и ...

Нихуя. Никакими десятками секунд и мегабайтами оперативной памяти и не пахнет (как в Experimental Results), алгоритм жрет всю память, пока не кончится. Опять же, я уверен, что так и должно быть, по мне дерево возможных вариантов анализа растет по экспоненте, а результаты предыдущих итераций хоть как-то, да будут учитываться.

Есть ли смысл писать статью-опровержение? Ведь подход, описанный в статье, так и не стал применяться и при рождении был заброшен. Я же могу указать, что он, в принципе, не имеет смысла. Примут ли такую статью от ноунейма? Как вообще устроен механизм опровержения выводов статей, есть ли какие-то научные комиссии из авторитетов?
399 482875
>>2873

>Есть ли смысл писать статью-опровержение?


Да.
400 482885
>>2870

>но чистые математики не могут в физику, совсем.


Архив в последние годы говорит об обратном, вывод - ты пиздобол.

>по физики


Сначала сделай домашку по русскому языку, а потом принимайся за взрослые вещи.

>да и годного учебного пособия по классической теории поля нет


Это потому, что ты ни хуя в теме не разбираешься. Богуш/Мороз.
401 482886
не знаю куда спросить
есть пластик доска для резки еды (мб кто знает хим состав?)
нужно вещество которое разьедает этот пластик или позволяет отделить от метала (да они сплавились)
402 482892
>>2886
Если пластик дешевый, то смываешь его растворителем - хлороформ/хлористый метилен/ацетон/толуол. Сделана эта доска скорее всего из какого-нибудь ПВХ или полистирола, вообще смотри маркировку на доске, а дальше гугли, чем растворять этот пластик. Моя доска, например, сделана из PE-LLD, и растворяется она, по утверждению википедии, в кипящем циклогексане или четыреххлористом углероде, либо в воде при 180 градусах. Можешь, конечно, поэкспериментировать с растворителями из хозмага, но я бы на твоем месте забил на это дело и отодрал пластик физическими методами, потому что кипящий циклогексан в условиях квартиры - это нерациональное со всех сторон мероприятие.
403 482893
>>2892
он втруднодоступных местах, попробую погуглить ещё
404 482894
>>2893
Ну в труднодоступных местах физическими методами тоже можно достать, расплавь/сожги этот пластик, а железка останется тебе. Но тогда есть вероятность испортить металл.
405 482896
>>2894
кстати об этом, я могу сжечь расплавить, но от этого останется чёрная поверхность на метале, я и вонь я не хочу чтоб метал вонял (круглая хуйня от газ плиты) не спрашивай как туда пластик залился
впринципе можно кипятком отмыть?
406 482898
>>2896
Кек, ну если эту штуку можно снять с плиты и кинуть в кастрюлю с кипятком, то можешь попробовать, но пластик скорее просто размягчится чуть-чуть, и его все равно придется чем-то отковыривать. А черную смолу с поверхности после сжигания можно как раз растворителем оттереть, потому что это уже не полимер, а какие-то продукты его горения, имеющие меньшую массу но это не точно. Можешь проверить, если доска осталась - сжигаешь ее кусок на поверхности какого-нибудь металла, потом пытаешься смолу оттереть растворителем из хозмага. Сработает - проворачиваешь то же самое с конфоркой, или что у тебя там.
15677274784881.jpg41 Кб, 480x497
407 482911
Человек очень выносливое животное, он может вставать в семь утра и разгружать вагоны целый день, кто так работал знает насколько тяжелый труд. Для сравнения гепард быстро бегает но на короткие дистанции, выносливость у него низкая. Или взять очень сильную гориллу, разгружать вагоны за бананы ее никак не заставишь, она скорее сдохнет чем согласится. Даже сидеть целый день за пекой с красными глазами и кодить, это очень большая концентрация внимания и расход энергии.
Вопрос: Есть в ли животном мире подобные примеры выносливости?Насекомых просьба не брать, слишком отличаются.
408 482914
>>2911
Нет, люди самые опущи
15541473421120.jpg5 Кб, 183x179
409 482917
>>2914

>Нет, люди самые опущи

410 482937
>>2917
А блин, точно, спасибо, вспомнил лошадок.

А ещё ослы и верблюды.

Люди наверное всё-таки больше потребляют с результатов своего труда чем эти, в среднем.
411 482938
>>2937
Поясните за ослов, они правда выносливее лошадей?
412 482961
Почему только Илон Маск догадался сажать ракеты? Тормозить двигателями, развернувшись задом наперед, умели ведь давно. Да и перед приземлением грузов ведь используют тормозные двигатели.
413 482965
>>2961
Невыгодна было, проще разъебать эту бочку где нибудь в море, чем ловить и тестировать, переживет ли она еще полет.
Маск посчитал, подужался и решил что надо попробовать и поиметь выгоду. Получается или нет, есть ли смысл это делать, время покажет.
414 482966
>>2961

>Почему только Илон Маск догадался сажать ракеты?


Потому что Маша популистка и выпендрёжница, которой нужно поставить галочку в бизнес-плане. Никакого практического значения посадка ракеты не несёт: это отработанная руина, её нельзя использовать повторно.
415 482968
>>2965

>Маск посчитал


Он считать умеет? За него другие умные дяденьки считали и проектировали. Маск денежки считает
416 482971
>>2966
Но ведь использовали повторно.
417 482972
>>2971
ряяя Машк мошеник ряяя я скозал!
418 482973
Почему болит голова если много думать?
419 482976
>>2971
Так это, Шаттлы тоже повторно использовали. Признали, что экономический просер и опасно. Что у Марии принципиально нового, ардуинка с гироскопами, я тебя умоляю.
420 482977
>>2966
Ага, частная компания, которая всё время оптимизирует каждый винтик по цене, не раз выкидывавшая или кардинально менявшая частично реализованные проекты, потому что так выгоднее, зачем то продолжает себе в убыток 3-й год дрочить реюз. А теперь ещё и ставит всё своё состояние на производство ракеты, которая должна достичь максимально невыгодного полного реюза. Поистине странная компания, экономит на всём и вся, а убыточный проект зачем то не отменили, хотя делали так несколько раз.
421 482979
>>2973
Болит не из-за этого, а из-за стресса, потому что занимаешься не тем чем хочется.
422 482980
>>2979
То есть меня заебла математика? Но как так? Я ощущаю, что она мне необходима в профессии, но че то сложновато как то уже и желание пропало. Как сделать так, чтобы она приносила удовольствие?
Другой анон
423 482981
Что лучше, письменные Фейнмановские лекции по физике или видео?
424 482982
>>2980

>необходима в профессии


Подсознательно ты не настолько говноед, как сознательно, в котором(сознании) тебя смогли промыть и навязать тебе делать говно которое не хочется за сомнительное вознаграждение.
425 482983
>>2980

>Как сделать так, чтобы она приносила удовольствие?


Чтобы стать говноедом нужно отупеть, настолько чтобы даже подсознательно не понимать что жрёшь говно.
426 482984
>>2983
Так мне раньше матеша нравилась, но в вузике сессии нанесли свою травму. Это не поедание говна. Это поедание мёда. Он сладкий и вкусный, но с какого то момента тебя этим мёдом перекормили. И вот ты хочешь получать от мёда такое же удовольствие, что и раньше, но уже не можешь.
427 482985
>>2984
То есть я хочу вновь научиться чувствовать вкус мёда.
428 482986
>>2985
И не блевать от него.
429 482988
>>2985
>>2986
Занимайся ей чисто для себя.

Но в таком случае ты скорее всего придёшь к тому что а нахуй на это вообще время тратить, если прибыли это не приносит.
430 482989
Может она тебе и не нравилась никогда, а ты просто был промыт. Типа что-то получалось и тебе приносило удовольствие из-за того что получалось, а не из-за того что это была математика.

Ну или реально нравилась, пока не задумался а нахуя это надо.
431 482992
>>1090 (OP)
https://www.youtube.com/watch?v=iq5DP4cJMDk
Что вы про это думаете? Пиздеж же, да?
432 482994
>>2992
Ну по химии к нему вопросов нет.Все реакции выглядят как должны выглядеть. Единственный вопрос, где он камень такой нашел.
433 482995
>>2994
Да просто под ногами. Судя по его каналу он вообще миллионером должен быть. Он там из любого мусора может золото добыть
435 482998
>>2996
Пирит
436 483000
Теория струн предполагает, что кварки напрямую обусловлены струнами или оставляют возможность для какой нибудь промежуточной хуйни? Как вообще по теории струн, на каждую элементарную частицу по вибрирующей струне?
437 483001
Теорию квантовой гравитации вообще подразумевает что гравитация это искривление пространства-времени, что нету никаких гравитонов, или если это так, и без них, то с ней ничего не выйдет?
438 483002
>>3000
зачем ты изучаешь хуету, ни одно предсказание которой не подтвердилось?
439 483003
>>2968
Ну дядьки, неважно, просто эксперимент поставили, вдруг выгорит. Если выгорит - Машк гений двигает технологии. Не выгорит - Ууу, тупой Машк, обосрался
440 483004
>>3001
Эти взгляды эквивалентны, пруфы: https://doi.org/10.1016/0003-4916(75)90302-4
Если у тебя есть обмен безмассовыми частицами со спиральностью +2/-2, то ото возникает автоматически
Абсолютно также, как живут вместе электромагнитное поле и обмен фотонами
441 483005
Как именно бактерии вредят организму?
Может быть вопрос тупой и простой, но гугл нихуя не смог дать ответ.
442 483007
>>3005
Бактерии производят бОльшую часть веществ, плавающих в нашем организме. Естественно, часть этих молекул неполезна (а остальная часть полезна)
Но даже самые злобные молекулы не вредят нам так сильно, как го-о, поставившее над проектом микробиом человека того же хуя, которого ставили над проектом геном человека, когда отжали его и хотели мурыжить двадцать лет воруя бюджеты и запутывая эту отрасль знания ложными докладами. Хорошо что человечество всё более забивает хуй на гос.институты и двигает науку частным образом.
443 483010
>>3007
Чем патогенные бактерии питаются в нашем организме? Не фаготицируют же они наши клетки? И как долго они живут без питания в воздухе в нормальных условиях, не превращаясь в споры?
444 483015
>>3010
Клетки любых многоклеточных окружены "жировым" (там сложный состав, но в основном липиды) пузырем, в добавок на мембранах наших клеток торчат различные сигнальные белки с углеводными цепями, которые можно легко оторвать и схавать - сахар же.
Вот так на нас целая кормовая база. В добавок, когда клетки запускает апоптоз, она сама разбирается на сройматериалы, которые для чужих клеток халява.
Но это касаемо бактерий, который живут на слизистых и коже.
В кишечники бактерии жрут пищу, которые мы когда-то съели, а наш организм уже за ними фактически доедает объедки.

>И как долго они живут без питания в воздухе в нормальных условиях, не превращаясь в споры?


Бактерий много видов и не все образуют споры. О бактериях лучше говорить в рамках колоний или популяций, а не отдельных организмов. Отдельные товарищи довольно дохлые, но вот колонии супер живучи, в добавок быстро приспосабливаются за некоторое время к изменениям среды. У них эволюция происходить считай на глазах, и они могут ебаться посредством горизонтального объема генов. А еще они могут в частичное наследование приобретенных признаков. Так что бактерии их ореал обитания гораздо шире, чем на тушки.

Однако микроорганизмы не ограничиваются бактериями, есть всякие грибки и прочие многоклеточные.
445 483016
>>3015

> Клетки любых многоклеточных окружены "жировым" (там сложный состав, но в основном липиды) пузырем, в добавок на мембранах наших клеток торчат различные сигнальные белки с углеводными цепями, которые можно легко оторвать и схавать - сахар же.


> Вот так на нас целая кормовая база. В добавок, когда клетки запускает апоптоз, она сама разбирается на сройматериалы, которые для чужих клеток халява.



И как вот это вот приводит к бактериальным заболеваниям?
446 483017
>>3016
Есть условные хорошие бактерии, у них есть негласных договор с нашим организмом. Наша иммунная система не ебашить их, а они не повреждают клетки, и не гадят едкими какашками и бздехами.
Однако колония бактерий может мутировать и послать нахуй договор, начал жрать, что не положено и гадить всякими ядами. Может даже хакнуть нашу сигнальную систему и заставить клетки самовыпиливаться, устраивая себе поток нямки.
Или такой сценарий, мы каким-то способом убили своих бактерий, а их место занимают всякие редиски, которым похуй договор, вот так начинается веселье. Поэтому нам нужна "нормальная" микрофлора с "договором", которая вытесняет собой охуевших пидоров.
447 483020
>>3017
Ты на микрофлоре зациклился, я говорю про такие бактерии, как тиф, чума, тубекулез и прочее. Они убивают организм из-за веществ, которые выделяют при метаболизме или есть какие-то еще способы?
448 483021
>>3020
Зависит от типа инфекции.
Кишечные как правило из-за продуктов метаболизмом патогенов + различные способы их самозащиты.
Всякие гнойные - это уже нас тупо жрут из внутри.
Туберкулез это вообще уберкомбо. Он не только в нас прорастает грибницу, да еще медленно травит и обманывает иммунную систему.
Я уже говорил, что некоторые бактерии могут хакать сигнальные системы, заставляя клетки организма самовыпиливаться.
449 483022
>>3021
Спасибо за ответы.
450 483026
>>3002
Да и не подтвердится никогда, это не значит что она бесполезна и с её помощью нельзя корректно промоделировать определённые процессы, даже если фундаментально никаких струн нет.
451 483029
Услыхал тут на стриме у Упоротого палеонтолога, что Савельева хотят принять в РАН. Правда что ли? Это же пиздец!!!
452 483030
>>3029
да похуй уже на них, пусть хоть собаку принимают
453 483034
>>3029

>Это же пиздец!!!


РАН пиздец ещё с 17 года. 1917 в смысле.
454 483039
Почему свет виден только когда падает на предмет? Если включить фонарик в темной пыльной комнате то мы увидим луч и пятно света на стене, а если из этой комнаты откачать воздух, пыль, небо и Аллаха, включить фонарик то луча мы не увидим. Только пятно света на стене. Фотон сам по себе не светит что ли?
image.png302 Кб, 640x425
455 483040
>>1277

>Блядь, ты слышишь меня?


Да да, я слушаю!
456 483047
>>3039
Чтоб увидеть фотон, он должен тебе попасть в глаз.
Когда включаешь фонарик, ты просто направляешь поток фотонов куда-то, но не в глаз. В глаз уже он попадет, когда он отразится от какой-нибудь хуйни и по стечению обстоятельств прилетит тебе в глаз.
Зашквар тоже переносится фотонами, так что можно зашкварится если на тебя упал зашкваренный фотон.
image.png13 Кб, 895x505
457 483048
>>3047
я тебя зашкварил
458 483049
Полистал

>Богуш/Мороз


(>>2885) увидел там вывод понятия энергии, пригляделся - обдристался.
У теоремы Нетер есть два (по крайней мере элементарных) подхода разных для энергии/импульса и для калибровочной инвариантности например. Поэтому некоторое время я даже не мог вдуплить в чем подвох - вроде помню какое то доказательство было, потом смотрю - хуякс а оно уже совсем другое.
Вообще теорема Нетер и формализмы Лагранжа и ко везде даются очень хуево. Потому что нихуя не понятно что переменная, что функция, что блядь варьируется что остается постоянным и почему. Авторы даже не объясняют что бывает пассивное, а бывает активное преобразование хотябы например. А потом у студентиков такая кашка в голове, и рождаются такие охуительные высеры вроде.
https://www.youtube.com/watch?v=YTUMDBOobTw
У Сассмена есть годный рант по этому поводу, так что со своим

>да ты просто тупой


тралята могут сразу нахуй пропутешествовать.
Если у кого есть прям сверх-годное описание теоремы Нетер и всего этого формализма, можно даже (и думаю даже лучше) с высоким уровнем абстракции - буду благодарен.
459 483051
>>3049
У Арнольда же в его Математических методов.
А вообще какие проблемы могут быть с Лагранжевой механикой? Почти везде сначала рассматривают движения в фазовом пространстве, обнаруживают, что они образуют группу, позже на свойстве групп определяют функционал и действие. По идеи студенты уже должны знать группы преобразований и элементы функциональщины и не должны заблудиться и путаться.
Теорема Нетер это уже нахождение инвариантов групп движений в фазовом пространстве.
460 483055
>>3051
Интересно как ты собрался "складывать" "движения". Ты тралишь что ли?
461 483056
>>3055
Лол.
Движение (R->TM) по определению гомеоморфизм. Композиция движений и есть "складывание" или "умножение".
Проблемы офицер?
462 483057
>>3056

>Движение (R->TM)


Не знаю почему ты определил движение именно так, но
берем композицию R->TM с R->TM. Ой что то типы не сходятся никак. Проблемы?
В math был чудик один который писал что физикам лагранжиан не нужен а нужна какая то ебанина неведомая вроде категорий пучков. Ты не он случайно?
463 483058
>>3049

>У Сассмена есть годный рант по этому поводу, так что со своим


А, так ты тот долбоёб, который кроме него ничего не читал/смотрел и кто срёт в каждом треде. Ясно, проходим мимо.
464 483059
>>3058
Только там совсем другой Сассххх был, а так думаю что ты угадал, можешь сосать хуй мимо.
465 483063
Почему частица и античастица аннигилируют с излучением, а частица и античастица возникающие в вакууме аннигилируют без излучения?
466 483072
Какая ртуть как жидкость? Густая как кисель? Как молоко? Насколько твёрдая (если плавать в ней) на примере известных бытовых жидклстей
467 483080
>>3063

>а частица и античастица возникающие в вакууме


Потому, что это не частицы, а виртуальные частицы.
468 483083
>>3080
То есть они анигилируют с виртуальным излучением?
469 483090
>>3072
Динамическая вязкость ртути 1.5 cП, вязкость воды 1 cП, вязкость молока 3 сП. То есть по вязкости это как вода, рукой не отличишь.
Коэффициент теплопроводности ртути 29.1, воды 0.6. То есть, опустив руку в ртуть комнатной температуры, она будет намного холоднее воды - тепло руки будет интенсивнее отдаваться ртути. Или другой пример, если вода ледяная (3 градуса С), то в ней можно некоторое время безопасно держать руку и её будет покалывать. Если ртуть будет ледяная, то скорее всего ощутишь сильную боль и резко её достанешь, либо произойдёт резкое обморожение.
Теперь о плотности ртути, которая равна 13,5 кг/л, против 1 кг/л у воды. Когда ты будешь стоять рядом с ванной ртути, и попробуешь в неё солдатиком опустить руку , то руку будет сильно выталкивать, ощутимо сильно. Примерно, сила выталкивания будет равна 13.5 кг за каждый килограмм твоей руки. Учитывая, что вес руки 4 кг, то выталкивать будет сила, пропорциональная 53 кг. Сам можешь представить, как это будет забавно смотреться. Если хочешь представить такую силу, то в качалке есть такое упражнение. Ну и для цифр, 53 кг - это 3 полных бутыля с водой для кулера.
Помимо этого, тут уже мои фантазии, у ртути большая плотность может привести к большой инерции. Что я хочу сказать - когда ты стоишь в реке и руками водишь под водой в стороны - ты двигаешь слои воды и чувствуешь сопротивление. А в ртути это сопротивление будет в 13 раз выше. И несмотря на то, что медленно ты сможешь водить руки практически без сопротивления, то как только ты сделаешь резкий взмах, то твои руки попытаются сдвинуть несколько тонн ртути, и ты не сможешь это сделать. По сути будет эффект, как в неньютоновской жидкости, но вызванный совершенно другой причиной - огромной плотностью.
Ну и ещё, на ртути можно лежать - в ней невозможно потонуть. Она не имеет запаха, обладает слабым металлическим привкусом.

инфа из интернета, погуглил для тебя, могу ошибаться, но старался проверять источники
image.png286 Кб, 604x453
470 483091
>>2841
Чтобы взорвать такой охуительно большой шар нужна бомбочка размером с песчинку на его поверхности. В чём ебанутость цифр?
471 483092
test
472 483122
>>3063

>Почему частица и античастица аннигилируют с излучением


Потому что закон сохранения энергии.

>а частица и античастица возникающие в вакууме аннигилируют без излучения?


Потому что хуйня к реальности отношения не имеющая.
473 483124
>>3122
Зачем тогда пиздеть что они появляются и исчезают?
474 483125
>>3124
Потому что это удобно.
475 483126
>>3125
Так а в чём смысл?
476 483127
>>3125
>>3122

>Потому что хуйня к реальности отношения не имеющая.


Кстати вчера смотрел лекцию где говорилось что они шевелят реальные атомы.
Пиздёж?
477 483128
>>3122

>Потому что закон сохранения энергии.


В чём у них несоответствие?
478 483134
>>3126
Рассмотрим самый простой случай.
У нас есть два поля электрон-позитронное и электромагнитное, они обедняются в одно общее поле. Позже это поле квантуется. Т.е берут все возможные состояния поля и приписывают этому состоянию комплексный вектор, который магией превращается в волновую функцию. На очень малых энергиях(обычно по ней квантуется) еще худо бедно можно подсчитать все возможные состояния поля, но когда энергии уже чуть больше, чем очень мало, то все состояния уже не описать через один квант. Вот тут начинаются проблемы, поэтому используют различные трюки, чтобы хоть как бы интегралы сходились. Одно из таких трюков условно считаем обычные частицы, но добавляем поправки на виртуальные частицы. В первую очередь смысл в том, что мы могли с приблизительно посчитать всю ебалу, чтоб теория была количественной и предсказательной.
479 483139
480 483174
Почему это законы физики для макромира якобы не работают для микромира или мегамира? Есть какие-то доказательства?
481 483181
>>3174
Какие законы, уточни.

Вообще скорее всего из-за квантования.

Кстати, поясните за квантование, откуда вучоные ТОЧНО уверены что именно у этой частицы вдруг неожиданно хватило энергии пройти через барьер эта частица туннелировалась через барьер, а не что пока она летела в неё въебала ещё одна чамтица, добавив энергии, или что преодолела барьер другая частица, случайно летевшая рядом, с бОльшей энергией?
482 483201
>>3127
Виртуальные частицы, это просто такие математический трюки, которые появляются, когда ты пользуешься некоторыми методами приближенного решения.
483 483210
>>3181
Знают, потому что теория предсказывает и это наблюдается. Если бы рассевание между частицами, а не туннелирование, то статистика была другой.
Но частицы даже по одной могут туннелировать.
Самый яркий пример туннелирования это ядерные реакции. Альфа частицы(и вообще любые заряженные частицы) при вылете имеют энергию меньше, если бы она просто вылетела прямо из ядра, вылетает как будто они были не в кулоновском поле.
484 483212
Математики, нужна помощь.
20 человек по очереди делают команду /random, которая генерирует число от 1 до 100.
У кого число больше, тот выиграл.

Какой шанс у человека выбить бОльшее число, чем остальные с первого раза?

Какой шанс получить ничейный результат? (к примеру, 18 человек выбили числа до 70, а двое - одинаковопо 88).

Ткните меня в какой-нибудь калькулятор или формулу.
485 483259
>>3212

> 20 человек по очереди делают команду /random, которая генерирует число от 1 до 100.


> У кого число больше, тот выиграл.


> Какой шанс у человека выбить бОльшее число, чем остальные с первого раза?


Давай подумаем. Короче у одного человека шанс выиграть 0.5. Вероятность победы = 0.5^20 = 9.53*10^-7
486 483260
>>3212

> Какой шанс получить ничейный результат? (к примеру, 18 человек выбили числа до 70, а двое - одинаковопо 88).


вероятность того, что человек выберет получит одно число 0.01. Вероятность того, что все остальные получат это же число = 0.01^20 = 1*10^-40
487 483261
>>3259
>>3260
>>3210

>Теорема умножения вероятностей для независимых событий P(AB) = P(A)*P(B) вероятность одновременного наступления двух независимых событий равна произведению вероятностей этих событий


>https://www.mathelp.spb.ru/book2/tv5.htm

488 483262
с вероятностями могу ошибаться, в универе 3 балла по матану, но вроде как правильно
489 483283
>>1090 (OP)
Привет, Анон. Я слышал что в этих ваших интернетах есть ресурс с бесплатными научными статьями и книгами. Подскажи его пожалуйста.
490 483288
>>3283
Ты хотел сказать пиратскими?
sci-hub
491 483289
>>3288
Нет, что ты, я не поддерживаю пиратство.
Спасибо
492 483290
>>3288

> пиратскими


>наука делается на деньги всех граждан


>50 долларов за прочтение статьи


что-то тут не так, согласны?
493 483291
>>3290
Кстати, умный вопрос, тупой ответ - почему научные статьи так дорого стоят? 50 $ за прочтение одной статьи. Вон, создательница сайхаба держит свой хостинг на донатах, и умещает все статьи человечества, какого хуя такие цены за статьи у журналов?
inb4: расходы на рецензентов - они работают забесплатно
494 483292
>>3291
Специально в этом вопросе не копался, так что однозначно ни фига утверждать не буду.
Но мне как-то думается, что основная причина - банальная жадность. Паблишерам хочется вкусно есть, жить в дорогих домах, ездить на крутых тачках и т.п. А денег никогда не бывает слишком много.
Если бы речь ещё шла об изданиях, ориентированных на широкого пользователя, выпускающихся и раскупающихся большими тиражами, то там издатели могли бы сделать цены пониже и взять количеством. Но научные статьи покупаются всё же ограниченным кругом.
Отсюда и цены.
495 483295
Если взять трансформатор и вынуть из него сердечник, то резко упадёт кпд. Куда уходит энергия?
sagiri3.jpg74 Кб, 700x393
496 483296
Призываю химиков ИТТ. Как заебашить свечу нищей чмохе в домашних условиях при отсутствии воска и парафина?
497 483297
>>3283
Киберчленинка
498 483298
>>3295
Излучение+нагрев воздуха (типа свч)
499 483301
>>3298

> Излучение


Что за излучение?

> +нагрев воздуха (типа свч)


Сверх высокие частоты на 50 герцах?
500 483302
>>3301

>Что за излучение?


Электромагнитное обычное. У тебя просто сердечник хорошо запирал электромагнитное поле.

>Сверх высокие частоты на 50 герцах?


Я же написал "типа". На частоте такой совсем чуточку (крайне милипиздрически) но ты греешь среду.
501 483311
Как добиться высокого вакуума в домашних условиях?
Можно ли сделать рабочую радиолампу с коэффициентом усиления хотя бы 25+ забив на вакуум?
502 483313
>>3311

> Как добиться высокого вакуума в домашних условиях?


думаю, никак. Если бы было наоборот, вакуумные насосы не стоили бы по 150к руб
506 483318
>>3313
А как же взять цилиндр и два хорошо шлифованых поршня, которые сводились бы так, что слипались, т.е. между ними не оставалось бы условно, ничего, и не развести их?
507 483319
>>3311

>рабочую радиолампу


Это такую в которой что-то бы нагревалось? О каком вакууме тогда может идти речь?
sage 508 483323
>>3318
ответил в перекате
509 483382
>>3311

>Можно ли сделать рабочую радиолампу с коэффициентом усиления хотя бы 25+ забив на вакуум?


Тиратрон
510 483391
Вообще никак не догоняю, почему в эксперименте квантового ластика измерение/стирание "холостых" фотонов меняет картинку на экране, которая была получена до того как "холостые" фотоны дойдут до наших детекторов.
sage 511 483493
>>3034
В 1917 наоборот попытались исправить пиздец длящийся 1000 лет
512 483685
>>3201
Не, типа серьезный дядька (на ютубе, лол) говорил что они шевелят реальные атомы, поэтому охладить их до близких к нулю температур не получается.
513 483704
>>3685
Ну пиздец у тебя аргументация. Да, кванты предсказывают это. Но они это и без виртуальных частиц предсказывают блять. Это как спорить, существует ли дискриминант.
Тред утонул или удален.
Это копия, сохраненная 18 ноября 2019 года.

Скачать тред: только с превью, с превью и прикрепленными файлами.
Второй вариант может долго скачиваться. Файлы будут только в живых или недавно утонувших тредах. Подробнее

Если вам полезен архив М.Двача, пожертвуйте на оплату сервера.
« /sci/В начало тредаВеб-версияНастройки
/a//b//mu//s//vg/Все доски